Evidence 1

Pataasin ang iyong marka sa homework at exams ngayon gamit ang Quizwiz!

Dana is on trial for killing her co-worker Harry. The prosecution's theory is that Harry and Dana went out on a date after work, and then went back together to Harry's apartment. At Harry's apartment, there was some type of argument, and Harry ended up dead with a stab wound in his chest. Dana concedes that she and Harry went out on a date after work, but claims that she left Harry after dinner and never went back to his apartment. The prosecutor wishes to call Detective Hansen, who will testify that Dana's fingerprints were found on a drinking glass in Harry's apartment. Detective Hansen will testify that he was on the scene looking for prints, found one on a drinking glass, and then sent the glass to Forensics-R-Us, a private, independent, forensic testing company. Detective Hansen also sent along a set of Dana's fingerprints, which he took from Dana as part of his investigation. Forensics-R-Us lifted the print from the glass and created a digital image of the glass fingerprint and of Dana's fingerprint and then isolated twenty different points of comparison on the glass fingerprint that were identical to twenty points of comparison on Dana's fingerprints. Forensics-R-Us sent the report to Detective Hansen. The Detective will testify that he read the report and concluded that the prints which were found on the drinking glass were Dana's. He will also testify that police detectives such as himself routinely rely upon outside forensic labs to lift the prints, process them, and compare them with the prints of known suspects. Is Detective Hansen's testimony admissible?

No. Detective Hansen cannot testify as to his opinion that the fingerprint at the scene matches Dana's print, nor may he explain the underlying data to the jury, because the underlying data is testimonial, and the individuals who created the underlying data are not available for cross-examination. Therefore, allowing Detective Hansen to testify would violate the Confrontation Clause. Correct. Although the detective's opinion and the underlying testimony may be admissible under Rule 703, the Confrontation Clause would bar this opinion. According to Williams v. Illinois, the underlying report would be testimonial because the private company reasonably knew it would be used at trial against a targeted individual. Therefore, the prosecutor cannot use the report or any opinion based on the report, unless the defendant has the ability to cross-examine the individuals who created the report.

Officer Daniels pulled Phillip over for speeding. Phillip's brother Tim was a passenger in the car. While Officer Daniels was writing the ticket, he grew suspicious based on how Phillip was acting. He asked Phillip for permission to search the car, and Phillip agreed. When Daniels opened the trunk, he saw a partially open gym bag and asked Phillip if he could look inside the bag. Phillip said "Yes, you can. You should know that this is my bag, but my brother Tim packed it and put it in there." Tim was still in the passenger seat at the time and did not hear the statement. Daniels opened the bag and looked inside and found six pounds of heroin. Daniels arrested both Phillip and Tim. Phillip and Tim were tried separately, and Tim went to trial first. At Tim's trial, the prosecutor sought to admit Phillip's statement to prove that Tim packed the bag. Is Phillip's statement admissible?

No. The statement is inadmissible. Correct. The statement is hearsay, and no exception exists to admit it. In addition, the statement is testimonial, and if Phillip does not testify in the case (as is likely, since he is facing his own criminal charges), the statement would violate Tim's Confrontation Clause rights under Crawford v. Washington.

Which of the following questions to a witness is MOST likely to be allowed?

On direct examination: "Isn't it true that you are the owner of Stevens Garage on 415 S. Main Street?" Correct. Although this is a leading question on direct, it is permissible to ask a leading question on direct on an uncontested topic such as basic pedigree information.

Martha is charged with defrauding a government contractor. The prosecutor has a character witness who will testify that he has worked with Martha for seven years and knows her reputation. He is willing to testify that Martha's reputation at work is that she is a very dishonest person. Under what conditions will the prosecutor's witness be allowed to testify?

Only after Martha testifies in her own defense OR calls her own character witness to testify that Martha is an honest person. Correct. The prosecutor's character witness can be used to attack Martha's credibility, or to rebut Martha's character witness. Thus, either Martha testifying or calling a character witness will open the door to allowing the prosecutor's character witness to testify.

Sandy is on trial for selling crack cocaine. To prove the crime, the prosecutor will call Officer Grange, an undercover officer who will testify that he approached Sandy and that she gave him a rock of crack cocaine inside of a red balloon. Sandy argues that she was not the one who sold Officer Grange the cocaine—she was merely standing next to the seller, and the police arrested the wrong person. The prosecutor also wants to call Officer Hansen, an undercover officer who purchased crack cocaine from Sandy one year ago. Sandy was arrested for that crime, pled guilty, and served six months in jail. Officer Hansen will testify that when he bought the crack cocaine from Sandy one year ago, she gave him the cocaine inside a red balloon. Is Officer Hansen's testimony admissible?

Only if the judge is convinced that using red balloons to package crack cocaine is a distinctive and unusual practice for drug dealers. Correct. This evidence is offered in order to prove identity under Rule 404(b)—that is, to prove that since Sandy was the one who sold drugs in this distinctive way one year ago, it is more likely that she was the one who sold drugs in this distinctive way in the current case. The prior act was only one year ago, and so it is close enough in time that it is not remote. This evidence has no probative value for this purpose unless the judge determines that packaging crack cocaine inside a red balloon is distinctive and unusual.

Wilbur is a defense witness in a criminal case. After Wilbur testified, defense rested and the plaintiff called Emma, who works with Wilbur in the same company. Emma testifies that she has known Wilbur for six years and worked with him on numerous projects. Emma testifies as follows: Plaintiff's Attorney: What is your opinion of Wilbur's honesty?Emma: I would never trust Wilbur. I think he lies whenever he sees a benefit to it. He is certainly the type of person who would lie on the stand if he saw something in it for him.Plaintiff's Attorney: Can you tell me why you have this opinion?Emma: Sure. Well, there are a lot of reasons. The first time we worked together on a project, his part of the project was late, and so the entire project missed its deadline by a week. I learned a few months later that Wilbur had gone to our boss afterward and told him that the delay was my fault, even though my part of the project was on time. I was furious. Is Emma's testimony admissible?

Only in part. Emma should be allowed to make the first general statement about Wilbur's character, but she should not be permitted to describe the episode with the delayed project. Correct. Rule 608(a) allows a character witness to give opinion or reputation evidence regarding the truthfulness of a fact witness. However, Rule 608(b) states that a character witness may not give specific act evidence to attack or support a witness' character for truthfulness.

Gary is suing Wendell's Hardware store, claiming that he purchased an industrial-strength power saw on January 24th, and that the saw broke down after only five minutes of operation. Wendell's Hardware claims that it never sold Gary the saw. In its case-in-chief, Wendell's calls its store manager, who will testify that the company regularly keeps records of every item that it sells and those records are kept in the regular course of business. He will then seek to admit the company's sales records for the entire month of January, in which there is no sale of an industrial-strength power saw. Gary objects to this evidence. How should the court rule?

Overrule the objection and allow the evidence. Correct. The fact that someone does not make a statement is usually not considered to be assertive conduct, since the "declarant" was not trying to communicate something with his non-statement. Even if it were considered to be assertive conduct, Rule 803(7) allows absence of business records to be admitted.

Harry is on trial for robbery. He calls Diane, a character witness, who testifies that she has known Harry for fifteen years and in her opinion he was an honest person who would never steal anything from anyone. On cross-examination, the prosecutor asks Diane: "Did you hear about the time that Harry was arrested for stealing a car two years ago?" Harry objects to the question. How should the judge rule on Harry's objection?

Overrule the objection and force Diane to answer. If the defense attorney requests, give a limiting instruction telling the jury that this is not meant to be evidence that Harry actually committed a prior theft, only a method of testing Diane's credibility and knowledge of Harry's character. Correct. This evidence is relevant to test Diane's credibility and knowledge of Harry's character. If she has not heard about the arrest, she does not know him very well. But if she knows about the arrest, her opinion about Harry having an honest character loses credibility. However, the limiting instruction is important to ensure the jury does not conclude that Harry actually did steal a car two years ago, since it is not admissible for that purpose.

James and Mike were stockbrokers who worked together at the same brokerage firm. James is now on trial for insider trading. The prosecutor wants to admit statements made by Mike which implicate James in the insider trading scheme. James objects, arguing that Mike's statements are hearsay. The prosecutor responds that although Mike's statements are hearsay, they should be admitted as an opposing party's statement because Mike and James were co-conspirators. The trial judge rules that Mike and James were co-conspirators and admits the evidence. James appeals the trial court's decision to admit Mike's statements. How should the appellate court rule?

Overturn the lower court ruling if the ruling was an abuse of discretion and affected James' substantial rights. Correct. Under Rule 103(a), an appellate court should overturn a lower court's ruling on an evidentiary issue if the ruling affected a substantial right of one of the parties, and as long as the party made a timely objection at the trial court level, the standard of review is abuse of discretion.

Which of the following opinion testimony would most likely require an expert witness in order to be admissible?

Percy was a witness to a vicious beating during which three police officers injured a suspect by kicking him and hitting him with nightsticks as he lay on the ground. Percy will testify that given the number of kicks and the severity of the blows, the suspect almost certainly suffered multiple broken ribs and permanent damage to his lungs and other internal organs. Correct. According to Rule 701, lay witnesses are permitted to testify about any opinion which is based on their own perception, helpful to the jury, and not based on scientific, technical, or other specialized knowledge. In this case, Percy is going beyond his own observations and giving an opinion about the effect of the blows on the suspect's body. There is no way he could arrive at a conclusion about broken ribs and permanent damage to organs without applying scientific or technical knowledge to his observations.

Thomas is on trial for killing his sister Jane. His defense is that Jane was actually killed by Jane's ex-husband Max. Max has left the country, and is currently living in Brazil, where he is beyond the jurisdiction of the court. However, Thomas explains that Max felt guilty about Thomas being prosecuted for the crime, so he sent Thomas a letter in which he confessed to the crime. The letter is properly authenticated as having been written by Max. Thomas seeks to admit this letter into evidence in order to prove that Max, and not Thomas, is guilty of killing Jane. Is the letter admissible?

Perhaps. It is admissible to prove Max is the killer, but only if there are corroborating circumstances that indicate that the letter is trustworthy. Correct. The statement may be admissible under Rule 804(b)(3) as a statement against interest, since it is a statement against the declarant's (Max's) interest and the declarant is unavailable. However, since the statement is being offered in a criminal case and exposes the declarant to criminal liability, Rule 804(b)(3)(B) states that in order to be admissible it must be supported by corroborating circumstances, such as some other evidence that Max committed the crime.

Which of the following would most likely be INADMISSIBLE?

Plaintiff is suing defendant for injuries sustained when the defendant hit her with his car. During settlement negotiations, the defendant admits that he may have been texting at the time of the accident, and he is willing to pay her half of what she is suing for to settle the case. Plaintiff refuses. During the defendant's case-in-chief, he testifies that he was not texting at the time of the accident. The plaintiff seeks to admit the statement defendant made during the settlement negotiations in order to impeach the defendant's testimony. Correct. Rule 408 bars all statements made during compromise negotiations if offered to prove or disprove the validity or amount of a disputed claim or to impeach by a prior inconsistent statement or contradiction. In this case the plaintiff is seeking to admit a protected statement in order to impeach the defendant with a prior inconsistent statement.

Which of the following is most likely to be admissible?

Plaintiff is suing the defendant in a civil suit alleging sexual harassment. The plaintiff alleges that the defendant, her supervisor, grabbed her buttocks and tried to kiss her in his office. The plaintiff offers the testimony of three women who will testify that when they worked for the defendant, he grabbed their buttocks or breasts in his office. Correct. Although Rule 404(a) states that evidence of prior conduct is generally not admissible to prove propensity in a civil case, Rule 415 states that any prior sexual assault is admissible in a civil case involving a party's "alleged sexual assault." This evidence may still be barred under Rule 403, but unlike the other options, it is not automatically barred by the character evidence rules.

Frank began digging a large hole in his backyard to install a small pond when he cut through an underground cable belonging to Starr Telephone. A Starr employee immediately came out to Frank's house to assess the damage, and he told Frank that the broken cable provided phone service to hundreds of businesses in the area, and having it broken was costing the company $10,000 per hour. It took the employee over four hours to fix the damaged cable. Two days later Frank got a letter from an attorney in Starr's office, demanding that he pay them $40,000 to reimburse them for the lost business from the broken cable. The letter threatened a lawsuit if Frank did not pay within thirty days. Frank immediately called the attorney on the telephone and admitted that he broke the cable and that he had not checked before digging to see if there was a cable. However, he argued that a $40,000 payment was unreasonable and he asked if they would agree not to sue if he paid them $5,000. Starr refused, and Starr ultimately sued Frank, seeking $40,000 in damages. At trial, Starr sought to admit the statement that Frank made over the phone to Starr's attorney in order to prove that Frank was liable in the case. Should the judge admit or preclude Frank's statement?

Preclude the entire statement. Correct. This entire statement is barred by Rule 408 as a compromise offer of a disputed claim when offered to prove liability or amount of damages. At the time the statement was made, Starr had made a claim for $40,000, and Frank's statement disputed the amount of that claim, so a "disputed claim" existed. Rule 408 bars not only the specific offer itself, but also any statement that the party made during the compromise negotiations, including admissions of liability.

Jonah is on trial for possession of cocaine. Jonah takes the stand in his own defense, claiming that the cocaine the police found in his car belonged to somebody else. Jonah has a prior conviction from five years ago for obstruction of justice, which is a felony. The elements of obstruction of justice are that an individual used violence or deception in order to impede a criminal investigation. The prosecutor seeks to admit Jonah's prior conviction to impeach him. What should the judge do?

Review the indictment from Jonah's obstruction of justice conviction to determine whether the acts he was charged with involved acts of dishonesty. If so, the crime is automatically admissible to impeach. If not, the judge should admit the prior conviction only if its probative value to impeach Jonah in this case outweighs the unfair prejudice to Jonah. Correct. Under Rule 609(a)(2), a prior conviction is a crime of falsity if the court can readily determine that establishing the elements of the crime required proving a dishonest act. A court is allowed to review the indictment to see if the defendant committed the crime with a dishonest act. If so, the prior conviction is automatically admissible to impeach. If not, the court should apply the balancing test from Rule 609(a)(1)(B), and admit the prior conviction only if its probative value to impeach Jonah in this case outweighs the unfair prejudice to Jonah.

Mortimer and Larry are both arrested for killing Mortimer's wife. The prosecutor's theory is that Mortimer wanted his wife dead so that he could inherit her money, and he hired Larry to kill her. After his arrest, Mortimer confesses to the crime, stating: "I needed my wife to be dead so I hired Larry to kill her for me." Larry makes no statements. The two defendants are tried together. Mortimer is not going to testify in the case. The prosecutor calls to the stand the police officer who interrogated Mortimer. The police officer is going to read Mortimer's confession to the jury. Larry objects. Out of the following options, which is the best action for the trial judge to take?

Rule that the statement will be precluded if hte two defendants are tried together because it is inadmissible against Larry and violates his Confrontation Clause rights, and the unfair prejudice against him cannot be cured with a limiting instruction. Correct. This is the holding of the Bruton case—that a co-defendant's confession cannot come in against a defendant, even if it is redacted or a limiting instruction is used—because the defendant has no way to cross-examine the co-defendant regarding that statement. Thus, if the prosecutor wants to use this statement against Mortimer, she must sever the trial and try Mortimer and Larry separately.

Theresa is on trial for aggravated assault. She has testified in her defense. She has three prior convictions: one for aggravated assault (a felony) from six years ago, one for fraudulently filing a false police report (a misdemeanor) three years ago, and one for selling narcotics (a felony) one year ago. The prosecutor seeks to admit all three prior convictions as evidence in order to impeach her through proof of character for dishonesty. Which convictions (if any) are admissible?

The aggravated assault is almost certainly inadmissible; filing a false police report is certainly admissible, and the selling of narcotics will be admissible if the probative value of the evidence in proving the defendant's propensity for dishonesty outweighs its prejudicial effect on the defendant. Correct. Rule 609 governs the admissibility of prior convictions when used to impeach witnesses. Felonies which are not crimes of falsity (such as the aggravated assault and the narcotics sales) are admissible if the probative value of the evidence in proving the defendant's propensity for dishonesty outweighs its unfair prejudice to the defendant. In the case of the aggravated assault, the prior conviction is almost certainly inadmissible, because its similarity to the current crime makes it extremely unfairly prejudicial to the defendant. The filing of a false police report is a crime of falsity, so it is automatically admissible under Rule 609(a)(2).

The plaintiff in a civil case called Oliver to testify. On cross-examination, the defendant's attorney asked Oliver if it was true that on September 15th of the previous year, he filed a fraudulent claim for insurance. Oliver denied that he did so. The defendant's attorney then admitted an affidavit signed by Oliver in which he admitted to the insurance company that his September 15th claim was fraudulent. The defendant's attorney admitted the affidavit to prove not only that Oliver filed the fraudulent claim (and thus had a propensity for dishonesty) but also to prove that Oliver had just lied to the jury when he denied filing it. Assume the plaintiff objected to all of the defendant's questions and actions during this time. At what point (if any) should the judge have sustained the plaintiff's objections and cut off this line of inquiry?

The defendant should not have been allowed to admit the affidavit, since it is extrinsic evidence. Correct. Rule 608(b) only allows an opposing party to ask questions about specific acts on cross-examination. The opposing party is not allowed to admit extrinsic evidence of those specific acts.

Simon is on trial for stealing a car. The car in question was a 2005 Honda Accord. In order to prove grand larceny, the prosecutor has to prove that the value of the item stolen is greater than $5,000. The prosecutor had the owner describe the car, and she said the car had 120,000 miles on it and that its paint was chipped and peeling. The prosecutor then called Jack, a used-car salesman. Jack has never had formal training as an automobile appraiser, but he has been in charge of setting prices for his dealership for twenty-five years. Jack had never seen the car in question, nor had he seen a picture of it or heard the owner describe it. The prosecutor asked Jack how much a 2005 Honda Accord would be worth, assuming it had 120,000 miles on it and that its paint was chipped and peeling. Jack replied that it would be worth between $4,000 and $5,000. The prosecutor then asked Jack how much it would be worth assuming it had only 50,000 miles on it and that its paint was perfect. Jack replied that it would be worth $10,000. What, if anything, is wrong with Jack's testimony?

The first question to Jack is appropriate, but the second question to Jack should probably not be allowed. Correct. Although experts can testify in response to hypothetical questions, most jurisdictions require that the hypothetical questions must either be based on facts that are supported by the evidence at trial, that can be inferred from evidence at trial, or that reasonably will be proven later at trial. There is no evidence (nor is there likely to be) that the car only had 50,000 miles on it. Also, this question is likely irrelevant in this case and may mislead the jury.

Thomas is suing PharmaGiant, a large drug company, arguing that PharmaGiant's new anti-cancer drug causes permanent memory loss and other brain damage in patients. As part of his case-in-chief, Thomas calls Dr. Kalas, who is an expert in epidemiology. Dr. Kalas testifies that he relied on data on brain damage gathered by independent doctors around the country and then processed the data to determine that brain damage was five times more common in patients who took PharmaGiant's new drug. PharmaGiant objects to this testimony, arguing that Dr. Kalas did not personally test the patients for brain damage; he relied on other doctors to measure the brain damage and then report the results to him. All parties agree that this data would be inadmissible for the truth of matter asserted under the hearsay rule. Thus, PharmaGiant argues that Dr. Kalas should not be able to testify about his conclusions. Should the court allow Dr. Kalas to testify about the underlying data and his conclusions?

The judge should determine whether the data gathered by the independent doctors on brain damage is of a type reasonably relied upon by experts in the field of epidemiology; if it is, the judge should allow Dr. Kalas to testify about his conclusions. Dr, Kalas can testify about the underlying data only if the judge determines that the probative value in helping the jury evaluate the opinion substantially outweighs the prejudicial effect of admitting the data. Correct. Rule 703 states that an expert may rely on the underlying data as long as it is the kind of data that is reasonably relied upon by experts in the field. Rule 104(a) states that a judge decides all preliminary questions about whether evidence is admissible, and this is a question of the admissibility of evidence. Rule 703 also requires the evidence to apply a balancing test before the underlying data can be disclosed to the jury, and if it is disclosed, it can only be used to assist the jury in evaluating the expert's opinion, not for the truth of the matter asserted.

Plaintiff is suing defendant, her next door neighbor, arguing that the defendant intentionally ran over her dog Fluffy with his car. The plaintiff claims that her dog was lying peacefully in her driveway, and the defendant intentionally turned in to run over the dog. The defendant claims that he was pulling into her driveway to turn around, and the dog ran in front of his car at the last second. The defendant testifies in the case. He cries during his testimony, saying: "I loved Fluffy. When I learned that I accidentally ran him over, I felt terrible." The plaintiff has evidence that the day before the defendant ran over the dog, he told his friend: "I hate our neighbor's dog Fluffy. The thing barks all night and keeps me awake. I wish it were dead." How (if at all) may the plaintiff admit the defendant's statement?

The plaintiff can ask the defendant about the prior statement on cross-examination, and can bring in the friend to testify about the statement. The statement is admissible to impeach the defendant and as evidence that the defendant actually hated the dog. Correct. This is a party-opponent statement, and therefore is exempted from the hearsay rule under Rule 801(d)(2). It is admissible for the truth of the matter asserted, and extrinsic evidence (such as the friend's testimony) is permitted. The statement also qualifies as a prior inconsistent statement to impeach under Rule 613, but Rule 613 only allows the plaintiff to use the statement to impeach.

Officer Ingram arrested Harry Pearson for disorderly conduct, and during the arrest Ingram broke Pearson's arm. The criminal charge against Pearson was dropped the next day, and the police department opened an investigation into Officer Ingram's conduct. Detective Graham of the internal affairs division ran the investigation. At the end of the investigation, Detective Graham filed a report concluding that Ingram failed to use proper police procedure and that he intentionally broke Pearson's arm because Pearson insulted him during the arrest. Pearson sued Officer Ingram for excessive use of force. As part of his case-in-chief, Pearson seeks to admit the results of the police investigation in order to prove that Ingram failed to use proper police procedure and that he intentionally broke Pearson's arm. Are the results of the investigation admissible?

The results of the investigation are admissible. Correct. The evidence is hearsay, but it is admissible under Rule 803(8), the public records exception. Rule 803(8)(A)(iii) creates a hearsay exception for records of a public office which are factual findings from a legally authorized investigation, and the results of the police investigation fit into that category. Results from an investigation are not admissible if they are offered against the defendant in criminal case, but this is a civil case.

George is suing Martha in a personal injury case, claiming that Martha's stairs were poorly maintained and therefore they crumbled when George stepped on them. One of George's witnesses is Frank, Martha's brother. During pre-trial discovery, Martha deposed Frank, and Frank said during the deposition: "About a week before George hurt himself, I saw that Martha's steps were rotting away. I told her to either get them fixed or to rope them off so that people wouldn't use them." George calls Frank at trial, but Frank refuses to testify against his sister, claiming "sibling privilege." The trial judge informs Frank that there is no such thing as sibling privilege, and that if he does not testify, he will be held in contempt. Frank still refuses to testify and is held in contempt and thrown in jail. George now seeks to admit Frank's statement during his deposition. Martha objects to the statement. How should the court rule?

The statement is admissible to prove the steps were rotting away and to prove Martha had notice they were rotting away. Correct. The statement is admissible under Rule 804(b)(1). Frank is unavailable under Rule 804(a)(2), since he refuses to testify about the subject matter even though he has been ordered by the court to do so. The rule states that testimony given at a deposition by a now-unavailable declarant is admissible if offered against the party who had an opportunity and similar motive to develop the testimony, as Martha did during the deposition.

Peter is suing Dr. Denard for malpractice. Peter had suffered from appendicitis, and Dr. Denard was the doctor who removed his appendix. Unfortunately, Dr. Denard left a surgical sponge inside of Peter's body, which resulted in severe complications. At trial, Dr. Denard claims that, consistent with standard medical practice, he relied on his nurse during the operation to keep track of how many sponges he had used. He testifies that as the operation was ending, he asked his nurse how many sponges he had put into Peter's body, and she had said "four." In fact, there had been five. Dr. Denard removed four sponges and closed the wound, inadvertently leaving a sponge inside of him. Peter objects to the part of Dr. Denard's testimony in which he testifies that the nurse answered "four" in response to his question. Peter argues this statement should be barred by the hearsay rule. The nurse has not testified in this case. Is Dr. Denard's statement admissible?

The statement is admissible, because it is not being offered for the truth of the matter asserted. Correct. The defendant is offering this statement not to prove the truth of the matter asserted (the truth of the matter asserted is that there were four sponges in Peter's body, but both parties concede that this was not true—thus, the defendant is clearly not trying to prove the truth of the matter asserted). Instead, the statement is relevant because it proves knowledge (or lack of knowledge) on the part of the defendant doctor—it is being offered merely to prove that the nurse said the word "four" and thus the doctor reasonably believed there were only four sponges left in the defendant's body.

One night the police respond to a 911 call and find Robert and Maggie, a married couple, having an argument. After interviewing both parties, the police arrest Robert and charge him with domestic violence, alleging that he hit Maggie in the face twice, causing physical injury. Under this jurisdiction, the crime of domestic violence includes an element of actual physical injury, not just pain. One week after the incident, Robert's defense attorney sends an investigator to Maggie's house. She tells him: "Robert didn't really hit me that hard; it hurt but there wasn't any bruise." The investigator also speaks to the arresting police officer, who tells the investigator: "Honestly I didn't see any bruise or any cuts; but the guy admitted to hitting his wife, so I arrested him." At trial, Maggie testifies that Robert hit her and created a large bruise on her face. The arresting officer has not had anything to do with the case since the arrest, and he does not testify at all at trial. Can Robert's defense attorney admit the statements her investigator elicited from Maggie and from the arresting officer?

The statement made by the police officer is inadmissible for any purpose, and the statement made by Maggie is only admissible to impeach Maggie and is not admissible for the truth of the matter asserted. Correct. Neither government agents nor victims count as "party-opponents" under Rule 801(d)(2). However, since Maggie testified in a manner inconsistent with this statement, the statement is admissible under Rule 613 to impeach her.

One day Debra, a four-year-old girl, disappeared from her front yard where she had been playing. A few hours later her family got a ransom demand from her kidnapper. After three days of negotiations, Debra was released in a nearby park in exchange for $10,000. Police officers investigating the crime developed probable cause to believe that Greg was the kidnapper. They arrested Greg and he was tried for kidnapping. Greg's defense was mistaken identity; he claimed he was not the kidnapper. The prosecutor did not want to put Debra on the stand because of her young age. However, Debra described to her father the location where she had been held for three days. She described a basement with a fish tank with large red fish, a Superman poster on the wall, and bright blue carpeting. The prosecutor wants to call the father to the stand to testify that Debra described the basement this way. The prosecutor will then call a police officer who will testify that he searched Greg's home, and that Greg has a basement with a fish tank with large red fish, a Superman poster on the wall, and bright blue carpeting. Should the judge allow the father to testify about what Debra told him?

Yes, because Debra's statements to the father are not hearsay. Correct. Debra's statements are being offered to prove that Debra has knowledge of what Greg's basement looked like. They are not being offered to prove the truth of the matter asserted—that is, they are not being offered to prove what Greg's basement looks like. The officer's testimony will establish that fact. Instead, they are being offered to prove that Debra has knowledge that would only be available to her if she had been kept in the room during her kidnapping.

Amy was robbed at gunpoint late one night, and the robber took her rings and her wallet. The police responded to the scene and took Amy to the precinct, where she examined hundreds of photos of possible suspects who fit her description. After about thirty minutes, she pointed at a photo of Zane Ferguson and said: "That's the man. I'm sure it was him." Zane was arrested and charged with armed robbery. Amy's wallet and rings were found in his home. At trial, Amy testifies and identifies Zane in court, and she testifies about selecting Zane's photo in the police precinct. The prosecutor calls the police officer, who will testify that Amy pointed at Zane's photo and said "That's the man." Zane objects to this testimony as hearsay. Should the officer's testimony be permitted?

Yes, because it was a prior statement of identification made by a testifying witness. Correct. Under Rule 801(d)(1)(C), any prior statement of identification made by a witness who has testified and was subject to cross-examination about the statement is exempt from the hearsay rule.

Gary was mugged on the street by two men with guns. He handed over his wallet and they ran away with it. Seconds later, in a state of great agitation, he called the police and reported the crime. In response to the questioning from the 911 operator, he told the police his location, a description of each of the robbers, and the direction in which they ran. The police caught the robbers ten minutes later and recovered Gary's wallet. The two suspects were charged with second degree robbery. A few weeks before trial, Gary was hit by a bus and killed. At trial, the prosecutor sought to admit his statements during the 911 call. Assume Gary's statements are properly authenticated. Are they admissible?

Yes, because they are excited utterances and they do not violate the Confrontation Clause because of the they are not testimonial if made during on ongoing emergency. Correct. The statements were made only a few seconds after a very traumatic event, and so the excited utterance exception applies. And under Michigan v. Bryant, the statements are not testimonial, because the information being conveyed to the police was necessary to combat an ongoing threat to the welfare of the community—two armed robbers were loose on the street, and Gary's information was critical in apprehending them.

The police exercised a search warrant on Carl and Stacy's apartment and they found half a kilo of cocaine hidden under the couch in a guest room. Carl and Stacy were both arrested. After the arrest, Carl maintained his innocence, claiming that he had no idea there was cocaine in the apartment. For her part, Stacy immediately told the police that the cocaine was hers and that Carl didn't know anything about it. The prosecutor did not believe that Carl was innocent, and she charged both Stacy and Carl with possession of a controlled substance with the intent to sell. Stacy pled guilty, but Carl went to trial. At trial, Carl took the stand and testified that he knew nothing about the cocaine, but that he had seen Stacy hiding something under the couch in the bedroom. He will further testify that she had told him not to worry about what it was and never to look at it, an order which he obeyed. In her rebuttal, the prosecutor offered evidence that Stacy had only a sixth grade education, that she had a very low I.Q., that she immigrated to this country only eighteen months before the police raided the apartment, and that she speaks almost no English. Carl objected to this evidence and was overruled. After the prosecutor's rebuttal, Carl responded by offering evidence that Stacy has two prior convictions for selling cocaine over the past eighteen months. The prosecutor objected to this and was overruled. Stacy did not testify in this case. Was the judge correct in making her rulings?

Yes, both pieces of evidence were properly admitted. Correct. The prosecutor's evidence of Stacy's education level, low I.Q., and proficiency in English is not technically character evidence; it tends to show that Stacy did not have the ability to buy and sell that quantity of drugs. Carl's evidence of Stacy's prior convictions could be considered character evidence and would be inadmissible if offered to prove Stacy had a propensity to sell drugs, but it is being offered here to prove knowledge opportunity under Rule 404(b)—that is, to show that Stacy did in fact have the knowledge and ability to sell drugs.

Betsy is suing her doctor, arguing that the doctor did not exercise the required level of care when he operated on her knee. Her husband testifies in her case, describing the difficulty that Betsy had moving around in the weeks after the surgery. At one point during his testimony, he testifies that "two months after the doctor performed the operation, Betsy could still not walk up the stairs of our home." The doctor's attorney cross-examines the husband and asks him whether it is true that two months after the operation, he had dinner with a friend and he said: "Betsy is really recovering well after her surgery. She can go up and down the stairs at our house with no problem." Can the doctor's attorney ask her this question?

Yes, but Betsy can request a limiting instruction which tells the jury to only consider the prior statement for impeachment purposes and not for the truth of the matter asserted. Correct. Rule 613 allows a party to impeach a witness with a prior inconsistent statement; this statement is inconsistent with her trial testimony. However, the statement is being offered to impeach the husband, not for the truth of the matter asserted, so a limiting instruction is required. This is not a collateral matter—that is, it is relevant to a material fact in the case (the complications that resulted from the surgery). Since it is not collateral, the doctor could call the friend to verify that the husband made the statement; he will not be stuck with the husband's answer.

Francine is suing her employer, Angler Markets, for wrongful termination. Angler's defense is that it fired Francine for cause, because of an incident in which she insulted a customer and used a racial slur against him. During its case-in-chief, Angler submits an e-mail written by the customer to the complaints department of the company. Francine's former manager will testify that gathering these complaints is a regular practice of the company, and that the complaints are gathered as part of a regularly conducted activity. She will state that because of these practices, she knows that the e-mail was received on July 5th, the day before Francine was fired. She will also authenticate the e-mail by confirming that the e-mail she has printed out is in fact the same e-mail that was received by the complaints department on July 5th. She will then read the e-mail, which states: "I was in your store yesterday and an employee named Francine was very rude to me. When I told her that I did not appreciate her tone, she made a denigrating comment about my ethnic background." Francine objects to the e-mail as hearsay. Is the e-mail admissible?

Yes, but only as evidence that the company had notice of the fact that a customer made a complaint about Francine, not as evidence that Francine did in fact insult a customer and make a denigrating comment about her ethnic background. Correct. The fact that the e-mail was received by the company is a business record under Rule 803(6), but the contents of the email ("I was in your store yesterday and an employee named Francine was very rude to me. When I told her that I did not appreciate her tone, she made a denigrating comment about my ethnic background") are double hearsay, and there is no exception to cover those contents.

Jarvis was driving home late at night when he struck Charles, who was on a bicycle. Charles was severely injured. The police were called and they gave Jarvis a Breathalyzer test. After he blew a .15—well over the legal limit—he was arrested and charged with driving while intoxicated. The next week Charles sued Jarvis, alleging that Jarvis was negligent, and seeking $100,000 in damages. One week before the criminal trial was set to begin, Jarvis offered to pay Charles $75,000 to settle the civil case. Will the prosecutor be able to admit the evidence of this offer in the criminal case against Jarvis?

Yes, if Jarvis told Charles that part of the conditions of the settlement would be that Charles refused to cooperate with the prosecutor in the criminal case. Correct. Rule 408 bars evidence of any settlement offer if offered to prove liability, but it specifically states that 408 does not apply if the evidence is offered to prove an effort to obstruct a criminal investigation or prosecution. Telling a witness in a criminal case that he cannot get a settlement unless he refuses to testify in a criminal case is an effort to obstruct the prosecution. This evidence is relevant to the criminal case against Jarvis because it tends to show he tried to obstruct the investigation because he believed himself to be guilty.

The state is prosecuting William for conspiracy to sell drugs. In its case-in-chief, the state offers the testimony of Raymond, one of William's former confederates who is now cooperating with the state. Raymond has a prior conviction for felony aggravated assault, which occurred eleven years ago. Raymond received probation for the crime. Can the defendant's attorney use that prior conviction to impeach Raymond?

Yes, if the trial judge finds that the probative value of the prior conviction substantially outweighs its prejudicial effect on the state. Correct. Under Rule 609, this is the test to use for admitting prior convictions that occurred over ten years ago, as is the case here.

Vector Industries is being sued by Giantsoft Inc. for patent infringement. Giantsoft claims that the computer code used on Vector's website is a patented piece of software that Giantsoft engineers wrote four years ago. Giantsoft wants to admit an email that Vector's CEO sent to David, his Vice-President for Technical Services. In the email, the CEO says: "David—Yesterday I spoke to Bill Donovan, our former webmaster, and now I know that last year he hacked into Giantsoft's website and stole all of their code. So it looks like all of our software was copied straight from Giantsoft's code. Call me as soon as possible." Bill Donovan had quit Vector six weeks before the email was sent. Is the email from Vector's CEO admissible?

Yes, it is admissible for the truth of the matter asserted—that is, that Bill hacked into Giantsoft's website and stole the code. Correct. The entire statement is admissible as a party-opponent statement. Although Bill was no longer an employee of Vector when he made the statement to the CEO, the CEO is a party-opponent and he adopted the statement as his own in the e-mail. A party-opponent need not have first-hand knowledge of the facts in his or her statement for the statement to be admissible under Rule 801(d)(2), as long as the party-opponent has adopted the facts in the statement as correct.

Tamara was arrested for domestic violence assault after she allegedly hit her husband in the face with a lamp. Before she was arrested, she told the police officers: "Sure I hit him. He had been yelling at me for twenty minutes, and I was afraid he was going to become violent, so I took matters into my own hands." At trial, the prosecutor seeks to admit her statement to the jury in order to prove that she hit her husband. Tamara's attorney objects that the statement is hearsay. Assume no Miranda issues apply. Is Tamara's statement admissible?

Yes, it is exempt from the hearsay rule as a party-opponent statement. Correct. Every statement made by an opposing party is exempt from the hearsay rule, whether or not she testifies and whether or not it was against her interest when she said it.

Carl is on trial for murder. He is pleading self-defense. The prosecutor has a character witness who will testify that in his opinion Carl is a vicious individual who routinely uses violence to get his way. Can the prosecutor's character witness testify at Carl's trial?

Yes, the character witness can testify, but only after the defendant calls his own character witness. Correct. Under Rule 404(a), the defendant "opens the door" to an attack on his character if he calls a positive character witness.

Frank is being prosecuted for armed robbery after allegedly holding up Debra in the street at gunpoint and stealing from her. The prosecutor calls Debra to the stand and asks her to tell the jury what was stolen from her during the incident. Debra says: "The guy took my watch and my wallet." The defense attorney has a copy of the police report, filled out by Officer Gendry, the officer who responded to the scene. On the report Officer Gendry wrote: "I spoke to Debra, the victim, while I was on the scene, and Debra told me that the perpetrator stole her wallet. I asked if the perpetrator took anything else, and Debra said no." On cross-examination, can the defense attorney ask Debra about the statement she made to the police officer in which she allegedly said that only her wallet and nothing else was taken? The defense attorney is offering this evidence to impeach Debra, not for the truth of the matter asserted.

Yes, the defense attorney can ask Debra about the statement she made to the police. Correct. Rule 613 allows an attorney to ask a witness about any prior inconsistent statement as long as the attorney has a good faith basis that the witness made the prior inconsistent statement. The description of Debra's statement on the police report gives the defense attorney a good faith basis.

Alex is on trial for burglary. The house he allegedly broke into had been for sale, and the prosecutor's theory is that a week before the burglary, Alex had shown up at the house as a potential buyer in order to learn the lay of the house and where it was vulnerable to break-ins. The defendant admits he came to look at the house as a potential buyer, but claims he was nowhere near the house when it was burgled. Alex has two prior convictions. One is for a felony burglary that occurred six years ago, in which he broke into a house that was for sale after he posed as a buyer and walked through the house three days before the crime. The other conviction is a misdemeanor for filing a false police report two years ago. Can she admit evidence of these prior crimes?

Yes, the felony burglary is admissible (subject to Rule 403) whether or not Alex testifies, but the filing of the false police report is admissible only if Alex testifies. Correct. The felony burglary is potentially admissible under Rule 404(b), because it tends to prove identity—that is, because Alex committed this crime in a distinctive way before, it is more likely that he is the one who committed the current crime. The judge will have to determine if the probative value of the evidence to prove identity is substantially outweighed by the unfair prejudice of the jury hearing that the defendant has a prior criminal record for burglary. As for the prior conviction for filing a false police report, it is automatically admissible as a crime of falsity under Rule 609, but only if Alex testifies, since its only probative value is to impeach Alex.

Sarah's boyfriend George was killed in his own kitchen with a knife early one morning. He had a knife in his hand. When the police responded to the call, they found Sarah in the kitchen crying, and the back door to the house was open. When the police asked Sarah what happened, she said nothing. The police continued their investigation over the next two weeks, and came to the conclusion that Sarah's grown son Andrew had killed George. The police arrested Andrew and charged him with murder. At trial, Andrew called his mother Sarah as a witness. She testified that she was the one who killed George, and that she acted in self-defense. She testified that George had come home drunk and began arguing with her and then grabbed a knife and swung it at her chest, and that she was able to grab her own knife and stab him in the chest. She also testified that she washed the knife she used and put it back in the drawer before the police arrived. She further testified that her son Andrew was never present that night. On cross-examination, the prosecutor asked Sarah if it was true that she said nothing when the police originally asked her what happened. Is this question permissible?

Yes, the question is admissible to impeach Sarah because silence in this context is inconsistent with her testimony of self-defense, since it would be natural for Sarah to have told the police about what happened when asked earlier. Correct. If an individual remained silent during a period of time when she would reasonably be expected to make a statement, under Rule 613 that silence can be interpreted as "inconsistent" with her current testimony that she was the one who killed the victim.

Jackson was pulled over after he was weaving across lanes of traffic. After observing him for a few minutes, the officer who pulled him over came to the conclusion that Jackson was drunk. Jackson refused to take a Breathalyzer test. The officer arrested Jackson for driving while intoxicated. After the arrest, the officer searched the car and found an empty bottle of wine and an empty paper bag. Inside the bag was an automated receipt from a liquor store that recorded a purchase of a bottle of wine with a credit card that matched Jackson's credit card number. The time of purchase listed on the receipt was 9:30 PM on that night. Jackson had been pulled over at 10:30 PM. At trial, the prosecutor wants to admit the receipt from the store to prove that Jackson bought the wine at 9:30 PM. Is the receipt admissible?

Yes, the receipt is admissible, but the prosecutor must first demonstrate that the machine that generated the receipt is reliable. Correct. The receipt is not hearsay, because any information generated solely by a machine is not hearsay. However, before information that is generated by a machine can be admitted, the party must establish that the machine is reliable.

Blackstone is a company which manufactures smart phones. Jonathon purchased a Blackstone phone and used it for two years, when he was diagnosed with brain cancer. He sued Blackstone, claiming that the radio waves emanating from his phone caused his cancer. In his case-in-chief, Jonathon calls Dr. Dark, who will testify that in his expert opinion the Blackstone phone can cause brain cancer. Dr. Dark has conducted numerous studies which have backed this up, and he has also spoken with dozens of other cancer specialists across the country, all of whom have reported a sharp increase in the instances of brain cancer in their patients who use Blackstone telephones. Dr. Dark testifies that in determining causation, doctors routinely rely on reports of correlation as reported by other doctors. Blackstone concedes that Dr. Dark is an expert who can give his opinion, but Blackstone objects to his testimony that dozens of other cancer specialists have reported the sharp increase in cancer for users of Blackstone telephones. Should Dr. Dark be permitted to testify about his conversations with the other specialists?

Yes, the statements are admissible, but only to assist the jury in evaluating Dr. Dark's expert opinion, and only if the trial judge concludes that the probative value in assisting the jury to evaluate his opinion substantially outweighs their prejudicial effect. Correct. The statements are hearsay, and therefore not admissible for the truth of the matter asserted (Rule 803(4) only covers statements that patients make for their own medical treatment or diagnosis, not statements that doctors make to each other). However, Rule 703 allows the expert to explain the data upon which he relied as long as it passes this balancing test.

Hawthorne Inc. is suing Draper Inc. for breach of contract. Hawthorne calls as a witness Stacy Williams, who works as a secretary in Hawthorne's office. Stacy will testify that on June 15th of last year, she was sitting at her desk next to another secretary, Heidi Logan. Stacy will testify that she saw Heidi talking on the phone and then hang up, and then Heidi turned to her and said: "That was Samantha, the secretary of the Vice-President of Draper. She just told me that the Vice-President told her he was not going ship the order to us because they found a buyer who will pay more money." Heidi is available but has not been called as a witness. Hawthorne has called Stacy to the stand to testify about what Heidi told her. Is Stacy's testimony admissible, and if so, for what purpose?

Yes, the testimony is admissible for impeachment and to prove that the Vice-President of Draper did not in fact ship the order to Hawthorne because they found a buyer who would pay more money. Correct. The Vice-President's statement is triple hearsay, but there is an exception for every level of the hearsay. The Vice-President's statement to his secretary is admissible as a party-opponent statement under Rule 801(d)(2)(C). The secretary's statement about what the Vice-President said is a party-opponent statement under Rule 801(d)(2)(D). And Heidi's report about what she just heard from Samantha is admissible as a present-sense impression under Rule 803(1), since Heidi is reporting about something she just perceived.

As part of a sting operation, the Capital City police department sent out an undercover officer to sell cocaine on the street. Thomas was walking by the undercover officer when the officer called him over and convinced him to buy some cocaine. After Thomas bought the cocaine, he was immediately arrested for possessing a controlled substance. At trial, Thomas is using an entrapment defense. Under the laws of his jurisdiction, this defense requires him to prove that he did not have any predisposition to commit the crime, and that the law enforcement officer persuaded him to take possession of the drug. In his defense, his calls his friend Terry. Terry will testify that a year ago he was at Thomas' house for a party, and Thomas noticed a number of guests snorting cocaine in the corner. Terry will further testify that Thomas became very angry, walked over to the group, and told them to stop and to never use cocaine in his house again. The prosecutor objects to Terry's testimony. Is it admissible?

Yes, the testimony is admissible. Correct. Although it is true that specific act evidence is inadmissible to prove propensity, in this case the evidence is being used to prove the defendant's character directly; i.e. that he did not have a predisposition to use drugs, which is relevant to Thomas' entrapment defense. Under Rule 405, specific act evidence is admissible if offered to prove character that is an essential element of a claim, charge, or defense.

Greg was suffering from depression, and his doctor proscribed Simtec, a new antidepressant developed by HealthFirst Pharmaceuticals. One year later, Greg had developed stomach cancer, and he sued HealthFirst, arguing that Simtec caused his stomach cancer. As part of his case, he calls his personal physician, Dr. Fleming, who diagnosed Greg's stomach cancer and predicts that Greg only has a few months to live. Dr. Fleming bases his opinion on a series of tests that hospital staff conducted on Greg at Dr. Fleming's request. Dr. Fleming himself did not conduct these tests, but he and other doctors routinely rely upon these sorts of tests in making a diagnosis, and the hospital routinely conducts these types of tests. Members of the hospital staff who conducted the test are not available to testify in the case. Dr. Fleming will testify on direct that in his expert opinion Greg has stomach cancer, but he will not discuss the tests that he reviewed or in any way explain the basis of his decision. The trial judge allows this. On cross-examination, HealthFirst seeks to admit the tests that Dr. Fleming relied upon, in order to show that the tests were flawed and insufficient to reach a diagnosis of stomach cancer, much less a prognosis of only a few months to live. The trial judge allows this as well. Was the trial judge correct?

Yes, the trial judge properly admitted both Dr. Fleming's opinion without any underlying data on direct, and the trial judge properly allowed HeathFirst to admit the underlying data on cross. Correct. Rule 703 allows an expert to testify to his or her opinion without revealing the underlying data, and Rule 705 states that the opposing party can always force an expert to disclose his or her underlying data on cross-examination. Finally, this is a civil case, and so the Confrontation Clause does not apply.

Egbert Oil runs a pipeline from the Gulf of Mexico into the Rocky Mountain region. On June 15th, the pipeline ruptured, spewing thousands of gallons of oil into the farmlands of eastern Colorado. The Colorado Environmental Agency ("CEA") immediately mobilized to contain the spill, and generated thousands of pages of mandated reports describing the spill and the damage done to the farmland. The CEA had never before responded to an oil spill; its usual purview was dealing with forest fires and pollution from factories; however, its statutory mandate included a requirement that it record and remediate any damage from oil spills. Dozens of landowners in Colorado are now suing Egbert, seeking restitution for the damage to their property. As part of their case, they are offering the CEA reports to prove the damage that was done to the land. Egbert objects to these reports as hearsay. Are the reports admissible?

Yes, they are admissible as public records. Correct. These documents are records of a public office concerning matters observed while under a legal duty to report, so they are admissible under Rule 803(8)(A) as a public record.

Stanley is accused of shooting and killing his wife. Stanley admits that he shot his wife, but claimed that it was an accident—that he had picked up the gun to see whether it was loaded and it went off. The prosecutor wishes to call Greg as one of her witnesses. Greg is one of Stanley's friends from the local bar, and Greg will testify that two weeks before Stanley shot his wife, Stanley offered Greg $1,000 if Greg would break into Stanley's house and kill his wife. Greg refused the offer, but never went to the police with the information until after Stanley's wife was killed. Stanley has not been charged with a crime for his offer to Greg. Can Greg testify about Stanley's offer?

Yes, to prove intent and lack of accident. Correct. Greg's testimony might have been admissible under Rule 404(b) even if Stanley were not claiming that the shooting was an accident, in order to prove that Stanley intended to kill his wife. Since Stanley is claiming that the shooting was an accident, it increases the probative value of Greg's testimony significantly in order to rebut Stanley's claim.

Corrine is suing Easy Cruisers, Inc. for a design defect in a yacht engine that Easy Cruisers built and sold to Corrine. During her case-in-chief, Corrine calls Karl to the stand as an expert witness. Karl is a high-school dropout with no formal training in boat design or mechanics. However, he runs his own marine engine repair shop, and has been personally fixing and rebuilding marine engines for over twenty-five years. Over the course of his career, he has repaired over a thousand marine engines for yachts. He will testify that he worked on Corrine's yacht engine after it broke down, and that in his expert opinion, the engine overheated and burned out most of the machinery because the oil line was improperly installed. Easy Cruisers objects to this testimony. Should the judge allow Karl's testimony?

Yes. Allow the testimony. Correct. Although Karl has not had any formal schooling or training, Rule 702 allows experts to be qualified because of their experience and skill, and Karl has had sufficient experience to be an expert in how marine motors operate. This is exactly the type of evidence for which an expert is required, so there is no concern about misleading a jury. And there is no rule which states that a plaintiff must wait to call an expert until after the defendant has called its own expert; on the contrary, the plaintiff has the burden of proof, and in a case like this, there may be no way of establishing the element of causation without calling an expert during the case-in-chief.

Debra is on trial for embezzlement after allegedly transferring six thousand dollars from a company account into her own personal account. The prosecutor has evidence that two years earlier, Debra stole two hundred dollars from her last employer by endorsing a check that was meant for the employer and depositing it into her own account. Will the prosecutor be able to admit this evidence?

Yes. If Debra calls a character witness who testifies that Debra has an honest character and is not the type of person who would steal, then the prosecutor can ask the character witness about the prior embezzlement on cross-examination of the character witness. Correct. If Debra calls a character witness who testifies as to Debra's honest character, Rule 405 allows the prosecutor to impeach that witness by asking her questions on cross-examination about Debra's specific acts if they are contrary to the character trait that the witness described.

Gary was bicycling through an intersection when a pickup truck driven by Karen struck him and killed him instantly. Wendy was in the passenger seat of Karen's car and observed the entire incident. Ten minutes after the event, the police interviewed Wendy, who was crying and visibly upset. Wendy told the police: "I don't know what the bicyclist was thinking. We had a green light and he just barreled across the intersection right in front of us!" Gary's family is now suing Karen. Wendy is out of the country on business, so Karen calls the police officer to testify as to what Wendy said about the light being green for Karen. The judge admits this evidence over Gary's objection. In his rebuttal case, Gary calls Wendy's friend Veronica. Veronica will testify that (1) Wendy and Karen are best friends, and have known each other since childhood; and (2) one week after the event, Wendy told Veronica that Karen ran a red light and killed a bicyclist. The judge allows this testimony over Karen's objection. Should the judge have admitted either the police officer's testimony or Veronica's testimony?

Yes. The police officer's testimony about what Wendy said is admissible, and all of Veronica's testimony is also admissible. Correct. Wendy's hearsay statement is admissible for the truth of the matter asserted as an excited utterance under Rule 803(2), and she can also be impeached under Rule 806. Note that Wendy's statements to Veronica are only admissible to impeach Wendy, not for the truth of the matter asserted. Crawford does not apply because this is a civil case.

Tracy is suing Windham Ladder Co., claiming that their ladder was manufactured with a design defect. She testifies on direct that the first time she used the ladder, it collapsed and she fell to the ground, breaking her ankle. Which question would most likely be permitted during cross-examination? (Assume that the opposing party has a good faith basis for believing that all of the facts being asked about are true).

"Isn't it true that you lied on your mortgage application when you tried to buy a house last year?" Correct. This is evidence of a prior dishonest action, and it is therefore relevant to Tracy's credibility and is admissible under Rule 608(b). The opposing party is allowed to ask the question, but she is stuck with the answer—that is, she cannot prove the false statement with extrinsic evidence.

Stephanie is suing Phillip for injuries she suffered when Phillip allegedly ran into her with a bicycle and then biked away. Phillip claims he was not even riding his bike that day and that someone else must have struck Stephanie with a bicycle. Stephanie calls Fran, an eyewitness, who will identify Phillip in court. In response, Phillip wishes to call Dr. Stevens to the stand. Dr. Stevens will testify that she is an experimental psychologist and has conducted dozens of tests and read about hundreds more on the topic of eyewitness identification. She will testify about the general unreliability of eyewitnesses, especially under stressful situations. She will also testify that she watched Fran testify and that based on her expertise Fran seemed to be unsure of her identification. What is the best objection that Stephanie can make to Dr. Stevens' testimony?

"Your Honor, Dr. Stevens' testimony will invade the province of the jury." Correct. Although most judges would allow an expert in eyewitness identification to testify generally about problems of reliability, almost no judge would allow her to testify that a specific witness was or was not credible, since credibility determinations are the province of the jury.

Which of the following opinions would most likely be admissible?

A narcotics detective with twelve years experience will testify that based on the amount of drugs recovered from the defendant and the way they were packaged, the defendant had an intent to sell the drugs. Correct. This witness is qualified to be an expert, and the testimony he is giving is well within his expertise. This testimony does not violate Rule 704's bar on opinions on an ultimate issue, because he is not basing his opinion on any knowledge about the defendant's mental state but instead based on the objective factors such as the amount and packaging of the drugs.

Which of the following evidence (if any) is barred by the hearsay rule?

A robbery victim testifies that he now has no memory of the robbery and cannot identify the defendant in court, but he testifies that in a line-up three days after the robbery he identified the defendant as the person who robbed him. After the robbery victim testifies and describes identifying the defendant in a line-up, the police officer who was present at the line-up also testifies that in a line-up three days after the robbery, the victim identified the defendant as the person who robbed him. A robbery victim testifies for the defense and states that the defendant is definitely not the person who robbed him. On cross-examination, the victim admits that he did indeed identify the defendant at a line-up three days after the robbery as the person who robbed him. None of the above; all of this testimony would be permissible under the hearsay rule. Correct. Rule 801(d)(1)(C) allows the witness to testify to any prior out-of-court statement of identification, even if it is inconsistent with the witness' trial testimony or if the witness now has no memory of the incident.

Oliver is an eyewitness to an armed robbery, and at trial he testifies for the prosecutor. On direct he testifies that he was about to enter his bank when he saw the defendant running out of the bank carrying a gun. He identifies the defendant in court. On cross-examination, the defense attorney asks Oliver if he ever filed a false insurance claim for his home insurance. This question is:

Admissible, as long as the defense attorney has a good faith basis for believing that Oliver has filed a false insurance claim. Correct. Rule 608(b) allows an opposing party to impeach a witness with a specific act of a prior dishonest action as long as the opposing party has a good faith belief that the act occurred. There is no requirement of notice. The question is not beyond the scope of Oliver's direct examination because impeachment is never beyond the scope under Rule 611. And the question is not irrelevant because a witness' credibility is always relevant.

After an argument about overtime pay, Cleon fired his secretary Fiona. Fiona called Cleon a few hours later and said that she had spoken to an attorney and that she intended to sue Cleon and the company for breach of contract. Cleon apologized over the phone and said that he had lost his temper. He then typed out a letter admitting that it was wrong to fire her, and offering to re-hire her and to pay her the overtime she had asked for. He then scanned the letter into his computer and sent her the letter attached to an e-mail. Fiona rejected his offer and sued Cleon, arguing that firing her and refusing to pay her overtime was a breach of their employment contract. She seeks to admit the letter as evidence that it was wrong for Cleon to fire her and that she deserved the overtime payments. The letter is:

Admissible. Correct. The letter is exempted from the hearsay rule because it is a statement made by a party-opponent. Although Rule 408 does bar settlement offers, it only applies to a "disputed claim." At the time that Cleon wrote the letter, he did not dispute anything that Fiona was alleging—in fact, he conceded that she should be re-hired and that she deserved the overtime payments. Thus, Rule 408 does not apply. Although Rule 1002 requires "an original writing," Rule 1003 makes it clear that a "duplicate is admissible to the same extent as the original" unless there is a genuine question about the original's authenticity or it would otherwise be unfair to admit the duplicate.

Detective Starr works as a narcotics detective for the Jonesville Police Department. He has been indicted on charges of theft and cocaine possession with intent to distribute. The prosecutor alleges that six months ago Detective Starr arrested a suspect for cocaine possession, and found five kilograms of cocaine in the trunk of the suspect's car, but only turned in four kilograms to the police property room as evidence, keeping one kilo for himself in order to sell. Detective Starr denies the charges, arguing that there were only four kilograms of cocaine in the trunk originally and that he never stole any cocaine. At trial, Starr calls two character witnesses. The first is Detective Reynoso, Starr's partner. Detective Reynoso will testify that he has worked with Starr for five years and that Starr is the bravest police officer he has ever known. The second witness is Sarah, Detective Starr's girlfriend. Sarah will testify that she has known Starr for ten years and that he is extremely honest and law-abiding. The prosecutor objects to both witnesses. How should the court rule?

Admit Sarah's testimony but not Detective Reynoso's testimony. Correct. Rule 404(a)(2) allows a criminal defendant to admit propensity evidence for any "pertinent trait," and Sarah's testimony that Starr is honest and law-abiding tends to prove that he did not commit the crime of stealing the cocaine with the intent to sell it. However, Detective Reynoso's testimony that Detective Starr is a brave a police officer is not pertinent to whether Starr stole the cocaine or possessed it with the intent to sell it.

Louis is on trial for robbing a convenience store. The prosecutor seeks to call his girlfriend as a witness to testify that Louis was a regular user of heroin, and spent over $50 on heroin every day. She will also testify that Louis owed his heroin dealer over $500. Louis objects to this testimony as inadmissible character evidence. The judge should:

Admit the testimony about Louis' heroin use and his debt to the dealer to prove motive. Correct. The evidence has significant probative value in proving that Louis had a motive to commit the crime, and it is admissible for that purpose under Rule 404(b). If Louis requests it, the judge will give a limiting instruction telling the jury not to consider the evidence for character purposes.

Frank and Allen ran a small software company together, and they developed a scheme to cheat on their taxes. They set up a dummy corporation, Alabaster, Inc., and they then made payments to Alabaster for "consulting" work. They deducted these payments from the reported income of their actual company and then paid taxes on the fraudulently lowered income. They then reported false "losses" for Alabaster so that Alabaster would have no income. Frank and Allen were ultimately arrested and their trial was severed. In Frank's trial, the prosecutor sought to admit an e-mail written by Allen to Frank which said: "I made another $50,000 payment to Alabaster today. That brings the total payments this year to $250,000." Frank objects to the e-mail as hearsay. Is Allen's statement admissible?

Allen's statement is admissible for the truth of the matter asserted as a party-opponent statement. Correct. Frank and Allen were co-conspirators, and Allen's statement was made during the conspiracy and in furtherance of the conspiracy. Thus, under Rule 801(d)(2)(E), these statements are admissible as party-opponent statements. A statement made in furtherance of the conspiracy is not testimonial, since the declarant did not reasonably expect it would be used at trial, so there is no Crawford problem with admitting this statement.

During a dispute in a bar, Charlie pulled out a knife and stabbed Harry in the chest. Harry died instantly. Charlie admits that he stabbed Harry in the chest, but he pleads self-defense, arguing that Harry swung at his face with a broken beer bottle first, and that he reasonably believed that he needed to use deadly force to protect himself. The prosecutor has a character witness who will say that Charlie has a reputation in the bar for being a very violent man. In his defense, Charlie wishes to testify that a few weeks before the incident, a friend of his had told him that Harry had stabbed two other people in the neck with a broken beer bottle in bar fights. How should the judge rule on the prosecutor's character witness and on Charlie's testimony?

Allow Charlie to testify about Harry's previous stabbing of two other people, but give a limiting instruction explaining that the jury should only use the testimony as evidence about Charlie's state of mind at the time of the stabbing, not as evidence of Harry's propensity for violence. The court should preclude the prosecutor from admitting evidence about Charlie's reputation for violence, either in the prosecutor's case-in-chief or in its rebuttal. Correct. Charlie's evidence about Harry's previous stabbings would be inadmissible to prove Harry's propensity for violence, because under Rule 405, propensity can only be proven by reputation or opinion evidence, not specific act evidence. However, because Charlie knew about these stabbings, it is relevant to prove that he acted with a reasonable belief that deadly force was necessary, and so it admissible for that non-propensity purpose. Because Charlie is not offering this as propensity evidence, he is not "opening the door" to prosecution propensity evidence, and the prosecutor cannot admit evidence of Charlie's propensity for violence.

Gary Smiles, a folk singer, is suing Amtrak Revival, a rock band, for copyright infringement. Smiles claims that Amtrak Revival stole the tune of one his songs "Forest Walk" and turned it into their number one hit "Burning Down the Forest." During the trial, Smiles wants to play his song "Forest Walk" for the jury and then play "Burning Down the Forest." In order for the jury to better hear the similarities, Smiles proposes that Forest Walk be played at twice the speed and in a slightly higher key than it was in the original, because then the similarities with "Burning Down the Forest" are more obvious. Amtrak Revival objects to this demonstration, arguing that instead Smiles should play "Forest Walk" at its normal speed and in its normal key. What should the judge do?

Allow Smiles to play "Forest Walk" at its normal speed and in its normal key and allow the jury to compare it with "Burning Down the Forest." Correct. This is relevant evidence. Smiles needs to prove to the jury that the two songs are similar, and in order to do this he must play the songs for the jury. However, allowing him to alter "Forest Walk" to make it sound more like "Burning Down the Forest" decreases the probative value of the evidence and increases the risk of unfair prejudice.

National Retail owns hundreds of warehouses across the country, where it stores merchandise for its discount retail stores. National Retail hired Cardez Shipping, a delivery company, to ship freight from its suppliers to its warehouses. After one year, National Retail began noticing that many of the boxes shipped by Cardez were arriving in the warehouses with significant damage. National Retail fired Cardez and sued the company for breach of contract under the Uniform Commercial Code. In its case-in-chief, National Retail called Zachary Winter, an industrial consultant who had worked for eleven years with companies in the shipping industry, advising them on how to run their businesses more efficiently. The Court certified Zachary as an expert in the shipping industry based on his education and experience. Zachary will testify to two points: (i) He will testify that he examined Cardez's quality control standards and that in his opinion they fell far below the industry standard for keeping freight safe and undamaged. (ii) He will further testify that this low level of quality control was a violation of at least three provisions of the Uniform Commercial Code, because his interpretation of the code was that, in the absence of any express contractual provision for quality control standards, the Uniform Commercial Code provided the applicable standard. Cardez objected to Zachary's testimony. How should the court rule on the admissibility of Zachary's testimony?

Allow Zachary to testify that Cardez's quality control standards were far below the industry standard, but do not allow him to testify that the Uniform Commercial Code applies and that Cardez's actions violate the Uniform Commercial Code, because that involves giving a legal opinion. Correct. Zachary is permitted to give his opinion on whether Cardez meets the industry standards—he is qualified to evaluate this issue, and it is a fact that helps the jury decide whether Cardez is liable in this case. Rule 704 also allows him to testify as to the "ultimate issue" in the case. However, he is not allowed to tell the jury that the Uniform Commercial Code applies, nor is he allowed to interpret the meaning of the Uniform Commercial Code—the job of interpreting the law belongs exclusively to the judge.

Gwen and her two children were flying on Patriot Airline, a charter plane company, when the plane crashed in the mountains. Gwen's husband sued Patriot Airline, alleging its pilot Ray was drunk when he was flying the plane and that he was improperly trained by his employer; thus, Gwen's husband alleges that the plane crashed because of pilot error. In its defense, Patriot Airline wants to call their Vice-President in charge of operations, who will testify about the extensive training and testing regimen that all of their pilots undergo before being entrusted to fly a plane. Patriot Airline also wants to admit evidence that the plane manufacturer who supplies planes to Patriot recalled its planes after the crash to fix an engine problem that tended to cause the motor to stall in cold mountain weather. Gwen objects to the Vice-President's testimony and to the testimony about the plane manufacturer's recall. Should the judge allow or preclude this evidence?

Allow both the Vice-President to testify, and allow evidence of the recall. Correct. The Vice-President's testimony is not about character and propensity; it describes the skills and standards that the airline imposes (and that Ray must have passed). Thus, this evidence is admissible to prove something other than propensity and is admissible under Rule 404(b). The evidence of the recall is relevant because it tends to show that the plane crashed because of mechanical issues, and not pilot error. Although the recall is a subsequent remedial measure, it is not barred by Rule 407 because it is a measure taken by a third party, and Rule 407 does not bar evidence of third-party subsequent remedial errors.

Sidney was charged with murdering his colleague Jake. The prosecutor's theory of the case is that Jake and Sidney had been embezzling money from the company for which they both worked. The prosecutor then alleges that Jake had been feeling remorse about the crime and told Sidney he was going to the police to tell them of the crimes. Sidney then allegedly killed Jake. At trial, the prosecutor seeks to admit evidence of the embezzlement that Sidney and Jake had been involved in. Sidney's attorney objects, arguing that Sidney is not charged with the embezzlement crime, so the jury should not hear about it. Should the judge allow or preclude evidence of the embezzlement?

Allow the evidence under Rule 404(b), because the evidence is not being offered to prove propensity to commit a crime but instead to prove Sidney's motive for committing the crime. Correct. The evidence has a high probative value to prove motive, and a very low level of unfair prejudice because the uncharged conduct of embezzlement is a very different kind of conduct than the homicide with which he is being charged.

Which of the following is likely to be INADMISSIBLE expert testimony?

An economist calculates the lost profit that a company suffered because of a trademark infringement. The economist relies in part on the statements made to him by the company president. The expert admits that economists almost never rely on statements made to them by corporate officers when making these estimates, but states that the business records alone were insufficient to draw a conclusion. Correct. Experts are permitted to use hearsay information as a basis for their conclusions, but Rule 703 says this is only permissible if experts in that particular field would "reasonably rely on those kinds of facts or data in forming an opinion on the subject."

Miles is on trial for running a dog fighting ring. The prosecutor alleges that Miles owned a compound outside the city where he trained dozens of dogs to attack other dogs. Miles would then allegedly host large dog-fighting parties at the compound, where he would force his dogs to fight each other in a large pit and charge his guests to watch. In his defense, Miles calls his wife Janet to the stand. Janet testifies that she has known Miles for five years, and that at home he never showed any cruelty to animals, and was always kind to his own pets. Assume that the prosecutor has a witness named Sarah who was at Miles' and Janet's house a year ago and saw Miles kick his pet dog in the head after the dog did not obey his command. Sarah also reports that Janet was present and saw Miles kick the dog. How (if at all) can the prosecutor use this information at trial?

Ask Janet on cross-examination: "Isn't it true that you once saw Miles kick your pet dog in the head when the dog did not obey his commands?" Correct. Although specific instances of conduct are not allowed to prove character on direct examination, they are permitted on cross-examination to challenge the opinion testimony of a character witness.

In a civil case, Brent wants to admit a statement made by Carl as a statement against interest. Brent has already established that the statement was so contrary to Carl's pecuniary interest that a reasonable person would not have made the statement unless he believed it to be true. What else does Brent need to establish in order to admit Carl's statement under this exception?

Carl is now unavailable to testify. Correct. Rule 804(b)(3), like all Rule 804 exceptions, requires that the declarant be unavailable to testify.

Charles is on trial for reckless driving. He is guilty of this crime if he knowingly drove fifteen miles an hour or more over the speed limit; he is accused of driving sixty-one miles an hour in a forty-mile an hour zone. His defense is that his speedometer was broken, and that therefore he did not know how fast he was going. He testifies in his own defense and tells the court that he conducted an experiment the day after arrest to determine whether the speedometer was working. He told his brother to get in his car and said: "Drive next to me, and as soon as your speedometer hits sixty-one miles an hour, stick your hand out the window." Charles then drove next to his brother on the highway, and when his brother stuck his hand out the window, Charles looked down at his own speedometer and saw that it read fifty miles per hour. Which of the following statements is true?

Charles' brother sticking his hand out the window was hearsay. Correct. When Charles stuck his hand out of the window, he was making a factual assertion; it was the equivalent of him yelling: "I am now driving at sixty-one miles an hour!" Therefore, his statement is an out-of-court statement offered to prove the truth of the matter asserted. The statement would almost certainly be admissible as a present sense impression, but the question asked about whether the statement was hearsay, not whether it would be admissible.

Samantha is suing her former employer, Dunaway Corporation, for age discrimination. Samantha alleges she was fired because Dunaway wanted their sales force to have a "younger look," and her manager Damon believed she was too old to work at the sales counter. Damon met with the Dunaway's Vice-President one week before she was fired, and Damon's secretary overheard the meeting while she was at her desk outside Damon's office. The secretary is willing to testify that she heard the Vice-President say: "Your saleswomen don't seem to match the image our company is trying to project," and Damon responded: "I know. I am going to let go some of the older saleswomen and I have some new college graduates I am interviewing for the open positions." Dunaway objects to this evidence. Damon is not a party to the lawsuit; Samantha is only suing Dunaway. Is Damon's statement admissible?

Damon's statement is admissible for the truth of the matter asserted as a party-opponent statement under Rule 801(d)(2). Correct. Although Damon himself is not a party to the lawsuit, he is an employee speaking within the scope of his employment, and therefore his statement is admissible as a party-opponent statement of Dunaway's under Rule 801(d)(2)(D).

Sandra was working as a welder for Ace Welding. After one year on the job, she began to suffer from severe headaches and blurred vision. She is now suing Ace Welding, claiming that the company did not provide her with the necessary safety equipment on the job. In particular, Sandra claims that Ace did not provide her with a breathing mask that would filter out the poisonous fumes that resulted from welding metal. At trial, Sandra called Dr. Gunn, who had a PhD in material science and metallurgy. Dr. Gunn was qualified by the court as an expert in metallurgy. Sandra's attorney then asked Dr. Gunn two hypothetical questions: If an alloy of iron and copper were heated to the point at which they became soft and molten, would any fumes emanate from the heated metal? And if an individual breathed in these fumes, could it cause her to suffer headaches and blurred vision? Should the trial judge allow these questions?

Dr. Gunn should be allowed to answer the first question regarding the fumes that emanate from heated metal, but not about the second question about headaches and blurred vision. Correct. Dr. Gunn is qualified to testify as an expert as to what happens to a certain kind of metal when it is heated, but he is not qualified to answer a question about how fumes could affect a person's health.

Fred is on trial for selling heroin. He has a prior conviction for burglary from two years ago. Under what conditions is it most likely that the prior conviction for burglary will be admitted in evidence?

If Fred testifies. Correct. If Fred testifies, he puts his credibility at issue in the case. He thus opens the door to the possibility that the prosecutor will impeach him with a prior conviction under Rule 609. The prior conviction still may not be admissible—the judge will have to determine that the probative value of the evidence for impeachment outweighs the unfair prejudice to the defendant—but at least there is a possibility that the burglary conviction will be admitted.

Greg is suing Veronica for breach of contract. Veronica has two prior convictions. Twelve years ago she was convicted of income tax fraud, a felony, and she served three years in prison. Six years ago she was convicted of filing a false police report, a misdemeanor, and she served six months in prison. Greg would like to admit the two prior convictions as evidence. Are they admissible?

If Veronica testifies, they are both automatically admissible. Correct. Under Rule 609(a)(2) any conviction for a crime of falsity that occurred within ten years is automatically admissible. The ten years is calculated from the time of "conviction, or release from confinement, whichever is later." Thus, the ten year time limit does not count any time spent in prison, so for the purposes of Rule 609, the income tax fraud conviction occurred nine years ago.

Sarah's father was ailing and moved in to live with her. In order to provide him with the proper care, Sarah signed a contract with Diane Watson, a private nurse, to stop by her house every day to check on her father's health and give him medication. After two weeks, Sarah's father had died, and she believed that Watson's poor nursing care was partially to blame. She sued Watson, alleging that Watson failed to detect serious health problems and failed to administer the proper medication to her father. In her case-in-chief, Sarah wants to call Dr. Taylor, a physician who used to work with Watson. Dr. Taylor will testify that six months ago, Watson gave one of his patients the improper dosage of medication for his diabetes, causing the patient to go into insulin shock and die. Dr. Taylor's testimony is:

Inadmissible under Rule 404 because it is propensity evidence. Correct. The only probative value of this evidence is to prove that Watson has a propensity to act in a careless manner and provide improper care, thus creating an inference that she acted in a careless manner and provided improper care when she was tending to Sarah's father. Thus, it is mere propensity evidence and barred by Rule 404. This evidence does not impeach Watson, since it does not contradict anything she testified to and is not a prior dishonest action. Thus, no exception applies to Rule 404, since this is a civil case.

Hydra Energy signed a contract with Daystar Natural Gas, by which Daystar agreed to provide Hydra with 10 billion cubic feet of natural gas per year in exchange for a specified payment. After the first year, Hydra claimed that Daystar had only provided it with 9 billion cubic feet of natural gas. Daystar claimed that it had provided the required 10 billion cubic feet, and that Hydra's storage facilities were so poor that it lost 10% of the gas that it stored. In its case in chief, Hydra seeks to admit two court judgments against Daystar—one from two years ago and one from five years ago. In each case, a jury found that Daystar had failed to deliver the agreed upon amount of natural gas to an energy company with which it had signed a contract. Daystar objects to this evidence. The evidence is:

Inadmissible. Correct. This evidence is best characterized as propensity evidence, since the only probative value of the prior contracts is to prove propensity. In a civil case, neither side can admit evidence of prior bad acts in order to prove propensity. Hydra may argue that the prior breaches of contract are habit, but two prior instances of breach of contract are not sufficient to establish a routine practice under Rule 406. Also, failing to deliver the required amount of natural gas is not specific enough to be considered routine practice.

Nick is on trial for killing Stanley. Nick admits that he killed Stanley, but he argues that the crime was involuntary manslaughter and not murder. Murder is defined as knowingly killing a human being. Manslaughter is defined as knowingly killing a human being under extreme emotional distress brought on by a provocation that would cause a reasonable person to feel such distress. Nick testifies at trial. He says that thirty minutes before the killing, a friend of his named Nancy approached him while he was at a restaurant and told him that she had just come from his house. She further explained that she, Stanley, and Nick's wife Nadine had been together at the house using cocaine, that Nadine and Stanley had gotten into an argument, that Stanley had started beating Nadine with a chair leg, and that Nadine was beaten so badly that she was taken to the hospital. Nick immediately left the restaurant and ran to his house, where he saw Stanley talking to a police officer outside his house. Nick then took out a switchblade and stabbed Stanley in the chest, killing him. The prosecutor objects to Nick's testimony. She agrees that Nick can testify that Nancy came into the restaurant and said something, and he can testify that he left the restaurant and went to his house and stabbed Stanley. However, she argues that anything that Nancy told him is hearsay and should be precluded. Are Nancy's statements admissible?

Nancy's statements to Nick are admissible because they are not being offered to prove the truth of the matter asserted. Correct. The hearsay rule does not bar these statements, since they are not being offered for the truth of the matter asserted. The jury does not really need to know whether Stanley actually did beat Nick's wife with a chair leg, only whether Nick believed that Stanley did so. Thus, the statements are relevant for a non-hearsay purpose: to demonstrate that Nick's state of mind was sufficient to deserve the mitigation to manslaughter.

Hank asked his friend Liam to invest in Hank's restaurant business. Liam agreed, and gave Hank $100,000. Six months later the restaurant failed, and the money was gone. Liam asked Hank for some of his money back, and Hank said the money was all lost in the failed business venture. Liam then sued Hank, seeking $50,000. At trial, Liam testified on direct that he and Hank had made an oral contract—Hank told him that if Liam gave him $100,000 and the restaurant failed, Hank would give Liam half of his money bank. On cross-examination, Hank's attorney asked Liam: "Isn't it true that after the restaurant failed, you simply made up this story about an oral contract in which you get half your money back?" Liam denied that he made the story up. On re-direct, Liam's attorney offers a letter written by Liam to his wife, dated on the day after Liam gave his money to Hank, in which Liam wrote: "I know this seems like a risky endeavor, but Hank has promised to give me half the money back if the restaurant falls through." Are there any hearsay problems with any of the evidence described in this scenario?

No, all of Liam's testimony is admissible. Correct. Testimony about a contract is never hearsay because the statements that make up the contract are legally operative statements; there is no "truth" being asserted. Also, Liam's letter to his wife is an out of court statement being offered to prove the truth of the matter asserted (that a contract was made), but Rule 801(d)(1)(B)(i) applies as an exception to the hearsay rule. This is a consistent prior statement by the witness which is being used to rebut an allegation of recent fabrication (the allegation made on cross-examination that Liam made up the story about the oral contract once the restaurant failed).

Sven was charged with possession of illegal drugs with the intent to sell. As part of her case in chief, the prosecutor offered a lab report prepared by a chemist who worked for a private laboratory. The lab report confirmed that the substance which had been recovered from Sven was LSD, an illegal drug. The actual chemist who performed the test was not available to testify. Instead the prosecutor called another chemist who worked in the same laboratory to authenticate the lab report. The witness will testify that the recovered substances are routinely sent to the private lab from the police in order to conduct these tests. She will further testify that these reports are generated in the regular course of business of the lab, and that making the record was a regular practice of the lab, and that the lab report was made by someone with knowledge at the time he made it. Is the lab report admissible to prove that the substance recovered from Sven was in fact LSD?

No, because admitting the lab report would violate Sven's rights under the Confrontation Clause. Correct. The lab report is testimonial, because the chemist who prepared it reasonably expected the information to used at trial. Thus, the defendant has a right to cross-examine the chemist and he does not have that ability at trial. The expert does not have an opinion independent of the private lab's analysis, so he cannot give an opinion without relying on the statements of others.

Michael is suing Dakota Pharmaceutical. Dakota manufactures and sells an anti-depressant medication called Celodex, and Michael took Celodex for six months and then developed stomach cancer. He claims that the Celodex caused his cancer, and at trial he intends to call an expert witness to prove causation. In its defense Dakota intends to call its own expert witness, who testifies that Celodex cannot possibly cause stomach cancer. The judge also decides to call an expert witness of her own. In order to ensure the expert remains neutral, the judge prohibits either party from deposing her expert before trial, though she does give each party a copy of the experts' report. The report concludes that Celodex cannot cause stomach cancer. Michael objects to the judge's witness. Did the judge follow the proper procedure?

No, because the parties have the right to depose the court-appointed expert. Correct. Rule 706(b)(2) states that parties have the right to depose all court-appointed experts.

Officer Joiner pulled over a car with two people inside: Avery and Bart. When the officer looked inside the car, he saw a large bag full of white powder sitting in the back seat. He asked both Avery (the driver) and Bart (the passenger) if either of them owned the bag, and both of them said they did not own the bag. The officer seized the bag and performed a field test which revealed that the white powder was cocaine. Avery and Bart were both arrested and charged with possession of cocaine. Avery pled guilty, but Bart went to trial. At Bart's trial, the prosecutor called Officer Joiner to the stand and asked him to testify about finding the cocaine. The prosecutor then sought to qualify Officer Joiner as an expert in narcotics laws. Officer Joiner had been a narcotics detective for twelve years, he had taken a six month class in the laws of narcotics in his state, and he had testified in over a hundred cases. Once Officer Joiner was certified as an expert, the prosecutor asked him to explain the doctrine of constructive possession. Officer Joiner correctly explained that under the laws of the state, all the occupants of a car possess every item in the car that is not found on someone's specific person. Officer Joiner then gave his opinion that based on where the cocaine was found, it was constructively possessed by both Avery and Bart. Bart's attorney objects to the testimony. Was the testimony proper?

No, no expert can ever give an opinion about domestic law, since that is the exclusive domain of the trial judge. Correct. Only the trial judge may give an opinion or an explanation that interprets domestic law.

Gary is on trial for robbing the First National Bank on December 10th. The prosecutor's theory is that Gary and his accomplice Bill robbed the bank together, wearing ski masks and driving away in the same car. Bill confessed to the robbery, and he has pled guilty and awaiting sentencing. He refuses to testify in Gary's case. The prosecutor seeks to admit Bill's confession. The confession does not mention Gary's name at all—Bill simply admits to having robbed the First National Bank with an accomplice on December 10th. The prosecutor has other evidence tying Gary to Bill—for example, Gary and Bill were together when they purchased ski masks the day before the robbery, and they were seen in a car together on the 10th only a few minutes before the robbery occurred. Gary's attorney objects to the confession. Is Bill's confession admissible?

No, the confession is not admissible because Gary has a Sixth Amendment right to cross-examine Bill and Bill is not available to be cross-examined. Correct. Under Crawford v. Washington, a criminal defendant has the right under the Sixth Amendment to cross-examine any witness who makes a "testimonial" statement against him. Although the Supreme Court has not settled on an exact definition of "testimonial," it certainly includes Bill's statement to the police about the crime, since Bill (the declarant) would reasonably expect this to be used at a future trial. Therefore Gary has a right to cross-examine Bill. Since Bill refuses to testify, his statements cannot be used against Gary.

Which of the following questions is least likely to be objectionable under Rule 611?

Defense counsel asks an eyewitness: "Isn't it true that you and the defendant have never liked each other?" Correct. This is a perfectly acceptable impeachment question, asking about bias against the defendant. A leading question is appropriate and permissible on cross-examination.

Which of the following pieces of evidence is most likely to be admissible to impeach the witness?

Fred is on trial for burglary. He does not testify, but Victoria testifies for the defense, telling the jury that Fred was with her all night at the night of the burglary and so could not have committed the crime. The prosecutor seeks to admit Victoria's prior conviction for selling cocaine, a felony, from five years ago. Correct. Under Rule 609(a)(1)(A), the appropriate standard for admitting a prior non-falsity felony conviction from within ten years for any witness other than a criminal defendant is the Rule 403 standard. This is the easiest standard from all of the options available.

Stuart is charged with stabbing a person in an alleyway outside a bar. He is pleading self-defense. During his defense case, his attorney has four character witnesses he wants to call. Which of them is most likely to be allowed to testify?

Stuart's father will testify that he has known Stuart his entire life and that Stuart has always been peaceful and never loses his temper. Correct. Stuart's peaceful character is a pertinent trait, and Rule 404(a) allows the defendant to elicit evidence that supports a pertinent character trait if given in the form of opinion or reputation evidence.

Blane is suing Taggert Industries for employment discrimination, alleging that Susan, Blane's supervisor and a Vice-President of the company, fired Blane because of his race. Blane calls Diedre as a witness. Diedre testifies on direct that she heard Susan, Blane's supervisor, using a racial epithet to refer to Blane a few weeks before Blane was fired. On cross-examination, Taggert's attorney asks Diedre if it is true that she told her friend a few weeks after the lawsuit was filed: "I am surprised; Susan has never done anything to make me think she was racist." Diedre denies having said this. Which of the following pieces of evidence (if any) could Taggert use to impeach Diedre?

Testimony from Diedre's friend that Diedre did in fact tell her "I am surprised; Susan has never done anything to make me think she was a racist." Correct. Rule 613 allows an opposing party to impeach a witness with any of her prior inconsistent statements. The opposing party cannot bring in extrinsic evidence of these statements if the statements are collateral (i.e., only admissible for impeachment purposes), but Diedre's statement about Susan not doing anything racist is not collateral—it is relevant to a fact in consequence. Thus, Taggert can admit extrinsic evidence (the friend's testimony) to prove that Diedre made the statement and thus impeach her testimony.

Which of the following out-of-court statements is most likely to be admissible in its entirety to prove the truth of the matter asserted?

"I am planning on going to school tomorrow to study geology." Correct. Rule 803(3) allows admission of a declarant's state of mind, including a statement about the declarant's future plans.

George is on trial for assault. The prosecutor argues that George got in a fight with a man outside a bar and beat him severely. George testifies in his own defense, and tells the jury that he was not present at the bar; he was across town at home eating dinner at the time the beating occurred. George then calls a character witness. Which of the following statements by the character witness would most likely be admissible?

"I have known George for ten years and in my opinion he would never be violent against anyone." Correct. Rule 404(a) allows a defendant to call a character witness to testify about a pertinent character trait of the defendant. Whether George has a violent propensity is a pertinent character trait.

Zach is on trial for killing Penelope. He calls Randy as a witness. Randy testifies that he was once roommates with a man named Gerald, and that Gerald told Randy that Gerald had killed Penelope. The prosecutor wants to object to Randy's testimony. Zach claims that Gerald's confession was a statement against interest, and that he has already tried to subpoena Gerald to testify, but that Gerald refused to testify based on the Fifth Amendment. What is the prosecutor's best response?

"Randy's testimony does not qualify as a statement against interest, because Zach has offered no evidence to corroborate the testimony." Correct. Under Rule 804(b)(3)(B), if a statement against interest is offered in a criminal case and tends to expose the declarant (in this case, Gerald) to criminal liability, the statement must be supported by corroborating circumstances.

Cameron died in her bed five years ago. The police investigated the case as a possible homicide. An autopsy was performed by the county coroner and the cause of death was deemed to be an overdose of heroin. The police ultimately decided that the death was accidental and dropped their investigation. However, recently the police began to suspect that Cameron's death was not accidental, but that Mindy, his girlfriend, intentionally injected him with a fatal dose of heroin because she was angry with him for being unfaithful. Cameron's body has been cremated, and the coroner who performed the autopsy has died. The prosecutor is seeking to prove the cause of death, and she offers the autopsy report conducted by the county coroner. The coroner is a government employee. What is the best objection that Mindy's attorney can make to keep the autopsy report out of evidence?

"The autopsy report is hearsay and there is no hearsay exception which will allow it to be admitted." Correct. The report is definitely hearsay (an out-of-court statement offered to prove the truth of the matter asserted), and it does not fit any hearsay exception. It may appear to be a public record under Rule 803(8), but this would count as "factual findings from a legally authorized investigation," and so it is inadmissible against a criminal defendant.

Victor is suing John for breach of contract. During the trial, Victor called two witnesses, Wendy and Diane, who testified as to the fact that John did not deliver the goods as promised in the contract. John did not cross-examine Wendy at that trial. The trial ended in a mistrial when three of the jurors got the stomach flu during deliberations after the judge had dismissed the alternates. Thus, the case had to be re-tried. In planning for the re-trial, Victor decided not to call Diane to the stand because she had proved to be a poor witness. Thus, he intended on going forward only with Wendy's testimony. Unfortunately, on the day of the re-trial, Wendy did not show up to testify. Victor had not issued a subpoena for her, because he had believed she would show up voluntarily as she had for the prior trial. Victor did not want to obtain a subpoena on the day of the re-trial, because that would require getting a continuance and he wanted the trial to take place on that day. Therefore, Victor sought to admit Wendy's testimony from the prior trial. What is John's BEST objection when confronted with Wendy's testimony from the prior trial?

"Your honor, this witness is not unavailable, and so her former testimony is not admissible under Rule 804(b)(1)." Correct. Rule 804(a) lists the different ways that a declarant can be deemed "unavailable" at trial, and it requires a party to use reasonable means to procure the declarant's attendance. Victor did not obtain a subpoena and he has not made any reasonable means to secure Wendy's attendance.

Sandy is arrested by the police for robbery. In which of the following situation is Sandy's confession LEAST likely to be admissible if offered by the prosecutor against Sandy? (Assume all Miranda rules have been complied with; i.e., there are no constitutional problems with any of the statements).

A few hours after the arrest, a police officer tells Sandy: "I just got off the phone with the prosecutor. She told me that she would be willing to charge you with a misdemeanor if you plead guilty at the arraignment and tell us right now the name of the person who committed this robbery with you." The police officer is lying—he has not spoken to the prosecutor—but there is no way for Sandy to know this. Sandy confesses to the robbery. Correct. Rule 410 will probably bar these statements. As long as the defendant reasonably believes she is engaging in a plea bargain, Rule 410 will protect his statements. In this case, although the police officer does not have the authority to make a bargain, the defendant does not know this, and his belief that he is plea bargaining is reasonable.

Which of the following is probably not a sufficient authentication?

A police officer who took a blood sample from the defendant is shown the vial with the blood sample and he confirms that it is the same blood that he took from the defendant. Correct. There is nothing in the fact pattern to indicate that the police officer sealed the vial and uniquely identified it, and there is nothing distinctive about the appearance of blood, so there is no way for the police officer to say with any credibility that this blood was the same blood he took from the defendant.

Which of the following is not self-authenticating?

A signed letter that is dated from twelve years ago, that transfers title of land. Correct. There is no rule that letters twelve years or older are self-authenticating, nor any rule that documents transferring title of land are self-authenticating. Under Rule 901(b)(8), an ancient documents can be authenticated by proving that it is at least twenty years old and that it is in a condition that creates no suspicion of its authenticity and a place where it is likely to be if authentic.

Dana is on trial for child abuse. The prosecution's theory is that on October 5th, Dana got angry at her six-year-old son for having stained the carpet in her bedroom, and so she struck him multiple times, causing significant bruising to his face and body. Which of the following evidence (if any) should be excluded as IRRELEVANT to the prosecutor's case?

A: One year ago, Dana's son was removed from her by the state for six months because Dana hit him in the face with a coffee mug and broke his jaw. B:Dana did not send her son to school for five days after October 5th. C: On October 12th, Dana's son told his teacher at school that a week before his mother had hit him and made him cry. D: All of the above facts are relevant to the case. Correct. All of the above facts are relevant because they have a tendency to make a material fact (that Dana did indeed strike her son) more probable than it would be without the evidence. Some of this evidence is barred as propensity evidence or by the hearsay rule, but the question asks about relevance, not admissibility.

Louis was pulled over for speeding by Officer Gareth of the Tarrytown Police Department. During this encounter, Officer Gareth allegedly ordered Louis out of the car and then struck him six times with his nightstick when Louis insulted him. Louis is now suing Officer Gareth and the Tarrytown Police Department, alleging that Officer Gareth used excessive force against him and that the Tarrytown police department failed to properly train, supervise, and discipline Officer Gareth. In his case-in-chief, Louis seeks to admit evidence that on three prior occasions in the past year, Officer Gareth had beaten motorists after he pulled them over for speeding. This evidence is:

Admissible against the Tarrytown Police Department, but not admissible against Officer Gareth if offered to prove he has a propensity for violence. Correct. If the evidence is offered to prove that Officer Gareth has acted violently in the past and is therefore more likely to have acted violently on this occasion, it is improper character evidence and is barred by Rule 404. However, it is relevant in the case against the police department as evidence that the department knew (or should have known) that Officer Gareth was violent and therefore should have taken some measures (training, discipline, termination) to prevent him from acting violently again. In other words, the police department could be liable for negligent entrustment by not taking action against Office Gareth. Thus, Officer Gareth's character is directly at issue in the case against the police department, and the evidence is not barred by Rule 404.

Jamie was opening a soda can when it exploded in her hands, causing serious injury. Her husband drove her to the hospital and dropped her off at the emergency room while he parked the car. She walked up to the desk and explained to the employee behind the desk that a soda can had exploded in her hands. Jamie later sued the company that manufactured the soda can, arguing that there was a defect in its design. During her case-in-chief, she called the hospital employee who admitted her to testify about what she (Jamie) said about the injury when she first arrived at the hospital. Jamie seeks to admit her statements to the employee to prove the truth of the matter asserted. Jamie's statements are:

Admissible if her statements were reasonably relevant to her diagnosis or treatment. Correct. These statements are likely admissible under the hearsay exception for medical diagnosis or treatment (Rule 803(4)). In order to meet the requirements of this exception, Jamie must prove that the statement was made for the purposes of medical diagnosis or treatment, is reasonably pertinent to that treatment, and describes her symptoms or the cause of her symptoms.

Harrison Oil runs ten large oil drilling rigs in the Gulf of Mexico. On July 12, a hurricane hit the region, and the high winds damaged one of the rigs, causing an explosion which killed two workers. The families of the workers are now suing Harrison Oil for wrongful death based on alleged negligent operation of the rigs. Harrison Oil kept the rigs running through the hurricane, even though the winds reached 85 miles per hour. One week after the hurricane, Harrison Oil changed its safety protocols so that it now shuts down its oil rigs anytime the winds get above hurricane force, around 74 miles per hour. During the plaintiffs' case-in-chief, the plaintiffs called an engineering expert who testified that the accident would not have occurred if Harrison Oil had shut their rig down before the hurricane and stored the more fragile equipment indoors. Another expert testified that the industry standard is to shut down oil rigs whenever winds reach hurricane force. During the defendant's case, Harrison Oil's Vice-President of Operations testified on direct that the company followed all necessary safety procedures during the hurricane and that the explosion was an unavoidable accident. He also testified that although it was common in the industry to shut down oil rigs during hurricanes, it would not make sense for Harrison to do so because it uses older model oil rigs which take six hours and dozens of employees to shut down the rig, which would endanger the employees' lives even more than if the rig were left running. The plaintiffs now wish to introduce evidence that Harrison Oil changed its safety protocols so that it now shuts down its oil rigs during hurricane force winds. This evidence will be:

Admissible only after the defendant's Vice-President has testified, and only to prove that it would be feasible for Harrison Oil to follow this procedure. Correct. Rule 407 allows evidence of subsequent remedial measures to prove feasibility if feasibility is controverted, and the defendant's testimony has controverted feasibility.

Julie Ann came home very late one night, and she was drunk, crying, and very upset. Her boyfriend Paul confronted her as soon as she got home, demanding to know where she had been. She started crying even harder, and explained that she was at a bar with some old college friends, and one of them, a man named Kyle, offered to drive her home at the end of the night. She agreed to go with him, but instead of taking her home, he drove her to an empty parking lot and raped her in his car. She said that he then drove her back to the bar and told her to get out of the car. She then walked ten blocks home. Kyle was arrested and charged with rape. At his trial, Julie Ann testified for the prosecution, and her story was consistent with what she told her boyfriend Paul on that night. The prosecutor then called Paul to the stand to report what Julie Ann told him when she got home. The defense attorney objects. Paul's testimony is:

Admissible under the excited utterance exception to the hearsay rule. Correct. The testimony is hearsay, because it is an out-of-court statement offered for the truth of the matter asserted (to prove that Julie Ann was raped by Kyle). However, at the time Julie Ann made the statement, she was "crying and upset," and so therefore in an excited state of mind, and the excitement was related to the subject of her statement (the rape).

Westin Coal owns and operates dozens of coal mines in the state. One day there was a cave-in on one of their mines, and three miners died. The miners' families are now suing Westin, arguing that the mines were negligently maintained. In their case-in-chief, the plaintiffs called a former Westin employee, who testified that no safety inspection had been conducted on the mine for over two years. During the defendant's case-in-chief, Westin called an employee from Prescott Insurance, a large national insurance company. The Prescott employee will testify that Westin has an insurance policy with Prescott, and that as part of that policy, Prescott employees conduct independent safety inspections on all of the Westin mines every three months. The Prescott employee will then authenticate and admit a business record from Prescott's files which confirms that Prescott conducted regular safety inspections up until the time of the cave-in. The plaintiffs object to the testimony of the Prescott employee. This evidence is:

Admissible. Correct. Although Rule 411 prohibits evidence that a party was or was not insured, it only prohibits the evidence if it is used to prove that the party acted negligently or otherwise wrongfully. Here, Westin is offering the evidence of insurance as proof that the safety inspections were in fact conducted.

Frank "The Snitch" Dunaway was arrested by an undercover officer when he tried to sell the officer five kilos of cocaine. As he was being arrested, Dunaway said: "You don't want me! You want my boss Victor! He gave me all these drugs to sell! I can tell you everything you need to know! Can we make a deal?" After Dunaway was arraigned, the officer took him to the prosecutor, where the prosecutor agreed to recommend probation if he pled guilty and testified against Victor. Dunaway then admitted to the crime and incriminated Victor in his statements. Later, pursuant to the agreement, Dunaway testified in front of the grand jury and admitted selling the drugs and incriminated Victor. When it came time to enter his plea, Dunaway refused to plead guilty, stating that he wanted to take the case to trial. At trial, the prosecutor sought to admit the following statements: (i) Dunaway's statements to the police officer. (ii) Dunaway's statements to the prosecutor. (iii) Dunaway's testimony in the grand jury. Should the court admit or preclude these statements?

Admit (i) and (iii), but preclude (ii). Correct. These statements are made by a party opponent, so they are exempted from the hearsay rule by Rule 801(d)(2). Rule 410 only precludes statements made during a plea bargaining session. When the defendant makes statements to the arresting police officer, he is not engaging in plea bargaining, since the officer has no authority to bargain. And when the defendant is testifying in front of the grand jury, he is not making statements during a plea bargaining session. However, the statements made to the prosecutor are inadmissible, since they were made during a plea bargaining session.

Robert was driving his pickup truck alone when he swerved to avoid an animal in the road. His truck hit a tree, and he was thrown from the truck and through the windshield. He was pronounced dead on the scene. Robert's wife Linda is now suing the truck manufacturer, claiming that Robert had buckled his seat belt, but that the seat belt was faulty and it unlatched when the truck hit the tree, causing Robert to fly out of the truck. The truck manufacturer claims that the seat belt was not faulty, but that Robert had not put on his seat belt for this trip. Linda will testify that she has ridden with Robert over a hundred times in his pickup truck over the past few years, and that he always puts on his seat belt. The defendant truck manufacturer objects to this testimony. Should the court admit or preclude Linda's testimony?

Admit Linda's testimony as habit evidence.Correct. Robert's tendency to wear his seatbelt qualifies as habit evidence under Rule 406, because it is a specific and repeated response to a specific situation, and it is semi-automatic behavior. It is relevant because it tends to prove that Robert did in fact wear his seat belt on the date of the accident.

Sam, Quentin, and Timothy were working together to sell cocaine. Timothy was arrested, and within a few hours he confessed, implicating Sam and Quentin. Sam and Quentin were arrested and charged with selling cocaine. Quentin was released on bail, and the next day he shot Timothy dead. He was caught in the act and pled guilty to the murder of Timothy and to the cocaine sales, stating that he and Sam had agreed that Timothy should be killed. However, he refused to testify at Sam's trial. Sam is now on trial. The prosecutor offers the statements that Timothy made during the confession which implicate Sam. Sam objects to this testimony. Should the court admit Timothy's statements?

Admit Timothy's statements, but only if the judge determines that Quentin killing Timothy was in furtherance of the conspiracy, within the scope of the conspiracy, and reasonably foreseeable as a natural consequence of the conspiracy. Correct. Under the forfeiture exception to the hearsay rule, Rule 804(b)(6), a defendant waives his rights under the hearsay rule if he acquiesces in wrongfully causing the declarant's absence. If a defendant's co-conspirator wrongfully causes the declarant's absence in a foreseeable way to further the conspiracy, the defendant has acquiesced in the wrongdoing. By doing so, the defendant also waives his Sixth Amendment right to cross-examine the declarant.

Coby was involved in an altercation outside the bar, and police were called to the scene. The police officers attempted to arrest Coby for disorderly conduct, but Coby became more belligerent and began attacking the police officers. The officers responded with force, and ultimately Coby was thrown to the ground, where he hit his head. The injury ended up being fatal.Coby's wife is now suing the police department, alleging improper use of force and seeking monetary damages for the death of her husband. Under the applicable law, Coby's wife must show that the police were unreasonable in their use of force and that their unreasonable use of force caused Coby's death. If she proves liability, Coby's wife can recover for emotional loss and lost earning potential.At trial, the plaintiff called an expert witness to testify as to Coby's future earning potential. The defendant police department then sought to admit evidence that Coby had been convicted of three different felonies, including once for selling drugs and twice for aggravated assault. Defendant police department argues that this evidence is relevant to show that Coby's earning potential is not as high as the plaintiff's expert claimed. Coby's wife, the plaintiff, objects to this evidence. What is the judge's best course of action?

Admit the evidence but sanitize it by only allowing the jury to hear that Coby had three felony convictions, but not the names of the crimes. Correct. The evidence has some level of unfair prejudice, but it also has a high probative value on the question of damages because a jury could find that a person who has three felony convictions has a lower earning potential than someone with no criminal record. Although the evidence may be admissible in its present form under Rule 403, the convictions for aggravated assault will improperly influence the jury into believing that the victim had a violent disposition, which is improper under Rule 404. This unfair prejudice can be mitigated by not revealing the names of the crimes to the jury.

Sam is charged with breaking into a safe in his supervisor's office in June and stealing merchandise that his supervisor stored there. Only two people—the supervisor and the private security officer who installed the safe—supposedly knew the combination to the safe. At trial, the supervisor wishes to testify that he believes Sam broke into the safe in April, two months before the incident in question. He thinks this is true because he came to work early one day and saw Sam in his office for no reason, and then he checked his safe later that day and items had been moved around. Because nothing was stolen, he never reported this incident to the police. The prosecutor argues that Sam's alleged entry into the safe in April is admissible under Rule 404(b), because it shows that Sam had the knowledge required to break into the safe (either he somehow had the combination or was able to crack the safe); and the fact that he had this knowledge makes it more likely that he broke into the safe in June. Sam's attorney concedes that if Sam had broken into the safe in April, the supervisor should be able to testify to that fact, but he argues that there is insufficient evidence that Sam did in fact break into the safe. Should the court admit the evidence that Sam allegedly broke into the safe in April?

Admit the evidence if the judge believes a reasonable jury could find by a preponderance of the evidence that Sam committed the prior crime. Correct. Sam's alleged prior crime is a question of conditional relevance, and so is governed by Rule 104(b). Although Rule 104(a) states that generally the question of admissibility depends on the judge's own decision, Rule 104(b) states that there need only be proof sufficient to support a finding that the fact does exist. The Huddleston case states that this means that there needs to be proof sufficient to support a finding by the jury that the fact does exist.

Gary and Charlotte went out on a date, and at the end of the date, they had sexual intercourse. Charlotte called the police the next morning and told them that Gary had raped her. When Gary was arrested, he claimed that the sex was consensual. Gary is now on trial for rape. Gary testifies that two nights before the alleged rape, he and Charlotte had gone out on a date which had ended in he and Charlotte kissing and engaging in other sexual activity short of intercourse. The prosecutor objects to this testimony. How should the judge rule?

Admit the evidence, because it is relevant to the case and there is an exception to the rape shield law for sexual conduct between the victim and the defendant. Correct. Rule 412(b)(1)(B) allows the defendant to offer evidence of the victim's sexual behavior with respect to the defendant if offered to prove consent.

Police had probable cause to believe that drugs were being stored in apartment 5D on the fifth floor of a certain building. They obtained a search warrant and searched the apartment, successfully finding a large quantity of heroin. However, there were no people in the apartment at the time, and the police were unable to determine who owned or lived in the apartment. The police then stationed themselves outside apartment 5D and waited. A teenager and his mother who lived in the building walked by and saw the police officers standing outside the apartment. The teenager turned to his mother and laughed, saying: "Those cops are morons. Big Jim isn't coming back to his place if there are a bunch of police waiting outside." A few minutes later an older woman walked by the apartment and the officer asked her: "Do you know where Big Jim lives?" and she pointed at the door of apartment 5D and then walked away. Police later arrested James Fanning for possession of heroin. At trial, the prosecutor established that in the neighborhood, Fanning is known as "Big Jim." The prosecutor then called the police officer to the stand, and asked him to testify about what the teenager said and what the older woman said. The evidence is being offered to prove that a person named "Big Jim" lived in apartment 5D. The defendant's attorney objected to this testimony as hearsay. How should the court rule?

Admit the statement by the teenager, but preclude the pointing by the older woman. Correct. The statement by the teenager was non-assertive conduct; that is, the information he meant to convey was that Big Jim would not return home, not that Big Jim did in fact live in the apartment. Thus, that statement is being offered not to prove the truth of the matter being asserted (that Big Jim would not return home), but to prove a fact that can be inferred from the fact that the declarant made the statement (that Big Jim lived in the apartment). The pointing by the older woman, although not an oral statement, was still a "statement" in the sense that it was intended to communicate a fact (that Big Jim lived in that apartment), and the prosecutor wants to admit this statement to prove the fact being asserted—thus, it is a hearsay statement.

Damon is on trial for vehicular manslaughter after he allegedly killed a ten-year-old child while driving. The prosecutor has three witnesses. The father of the victim will testify that he saw Damon swerve off the road and up onto the lawn, where he struck the child. The police officer who responded to the scene will testify that Damon was crying and distraught a few minutes after the incident, and said that he only looked away from the road for a second to check his cell phone. A coroner will also testify that he examined the child's body, and that the cause of death was massive trauma to the chest, likely caused by a car. Damon objects to the father's testimony as unfairly prejudicial. He notes that the father will no doubt be very emotional when he testifies about his son's death, and that this extreme amount of emotion will unfairly influence the jury. Damon is willing to agree to admit that he swerved off the road, up onto the lawn, and struck the child. Damon says that once he has admitted this fact, the father's testimony has little extra probative value and should be precluded. The prosecutor still wants the father to testify. What should the court do?

Allow the father to testify with no limiting instruction. Correct. Although there will be some level of unfair prejudice when the jury sees the father react emotionally on the stand, his testimony is extremely probative because he is an eyewitness to the event. Courts routinely allow victims and others with close relationships to the case testify even though they may display emotion; the unfair prejudice does not substantially outweigh the probative value of this testimony. Furthermore, in Old Chief v. United States (U.S. 1997), the Supreme Court stated that a party should only forced to accept an admission instead of offering live testimony if the probative value of the live testimony is extremely low.

Who of the following would be LEAST likely to be allowed to testify?

An eyewitness who suffered brain damage and now has no independent recollection of the subject matter of the testimony. However, a few weeks ago the eyewitness read his own diary entries about the event that he made immediately following the event, and he is now willing to testify about what he read in his own diary. Correct. A witness who has no memory of the event cannot testify about the event. Although hearsay exception 803(5) may allow him to read from his diary on the stand, he cannot testify about something he read a few weeks ago, even if it was his own diary entry.

Coding, Inc. is suing Icarus Software. Coding argues that the find-and-replace function of Icarus' new word processing program violates the patented find-and-replace function that Coding uses in its own word processing program. Icarus claims that its software does not violate the patent, and Icarus further claims that Coding's patent for the find-and-replace function is invalid because it patented a technique that was already in the public domain. Assume that the three elements to prove patent infringement under the federal patent act are (1) the plaintiff owns a patent; (2) the patent is valid; and (3) the defendant's product violates that patent. The federal patent act states that any party challenging the validity of a patent has the burden of persuasion and must prove the patent's invalidity by clear and convincing evidence. What is the best description of each party's burden of proof in the case?

As the plaintiff, Coding must prove by a preponderance of the evidence that it has a patent and that Icarus violated that patent. If Coding succeeds in proving these two facts, then Icarus must prove the patent is invalid by clear and convincing evidence. Correct. The presumption in the federal patent law controls, and therefore Coding has no burden of persuasion on the element of the patent's validity. However, as the plaintiff, Coding still has the burden of persuasion for every other element of its claim.

Bernard was on trial on ten counts of conspiracy to sell narcotics and for arranging the murder of an undercover police officer. The chief investigator in the case, Detective Mums, has been investigating the case for six months and has helped the prosecutor prepare and coordinate the seventeen witnesses necessary to prove their case. The prosecutor tells the court that the detective is essential to her presentation. Detective Mums also plans on testifying in the case herself. Bernard intends to testify in his own defense. He also intends on calling his brother as a character witness. Before the trial begins, the judge issues a ruling under Rule 615 to sequester the witnesses. Who of the following is required to leave the courtroom?

Bernard's brother. Correct. Rule 615 requires most witnesses to be excluded from the courtroom, and there is no exception for character witnesses or family members of parties.

Law enforcement officials searched Paul's computer pursuant to a warrant and found ten photographs containing child pornography. The officers seized Paul's computer and printed out copies of the images. Paul is now being prosecuted for possessing images of child pornography on his computer. At trial, the prosecutor does not admit the computer or the printed copies of the images. Instead, the prosecutor calls Officer Faraday, who conducted the search of Paul's computer and personally viewed the images. Officer Faraday will testify about the images of alleged child pornography that he saw on Paul's computer, describing the images in detail. What is the best objection that Paul can make to this testimony?

Best evidence rule. Correct. The prosecutor is trying to prove the contents of the photographs, and under Rule 1001, photographs are covered by the best evidence rule. Thus, a witness cannot testify to the contents of the photographs; instead, the photographs themselves (or duplicates, such as the printouts), must be admitted.

Stan is charged with felony domestic violence after he allegedly struck his girlfriend in the chest with a baseball bat. At trial, the prosecutor will seek to offer evidence that six months earlier, Stan had beaten his former girlfriend with his fists so severely that she went to the hospital for two days. Stan is pleading self-defense, and at trial he will seek to admit evidence that three years earlier, his girlfriend got into a dispute with a store manager while she was shopping and she pushed the manager into a glass table, causing multiple lacerations and requiring the manager to get over twenty stitches. There is no evidence that Stan knew about the incident with the store manager at the time he struck his girlfriend with the baseball bat. How should the judge rule on these proposed pieces of evidence?

Both pieces of evidence are inadmissible regardless of the order in which they are offered. Correct. Although Rule 404(a)(2) allows the criminal defendant to go first in offering propensity evidence of the victim, and then allows the prosecutor to respond with evidence of the defendant's propensity, Rule 405 states that when character evidence is offered to prove propensity (as here where Stan is proving that his girlfriend has a propensity to be violent, and therefore is more likely to have been acting violently on this occasion), character evidence can only be proven on direct with reputation or opinion evidence, not with specific instances of conduct. Stan could try to argue that his girlfriend's violent action towards the store manager is admissible to support his self-defense claim, to show he had reasonable fear of her when he hit her with the bat, but since he had no knowledge of his girlfriend's violent action at the time he struck her, this evidence is irrelevant to his state of mind at the time of the alleged crime.

Lisa owns a small antique store, and one night her store was burglarized. After the police arrested a suspect for the crime, she testified in the grand jury about all of the items that were stolen from her store. As part of her testimony, she read from her store's purchasing orders, which stated the amount of money that each item was worth. After the grand jury testimony, the prosecutor gave Lisa a copy of the transcript of her testimony, and she reviewed it and agreed that it was an accurate record of her testimony. A few weeks later, Lisa's store was destroyed in a fire. All of her records were lost, including the purchasing orders that she had read to the grand jury. When the burglary case went to trial, the prosecutor had to prove that the total value of the items stolen was over a certain amount. Unfortunately, Lisa cannot remember the value of each of the items. How can the prosecutor prove these values?

Call Lisa to the stand and ask her if she remembers the values of the items that were stolen. When she says she does not, ask her if the grand jury transcript is an accurate record of her testimony that she adopted at a time when it was fresh in her memory. When she agrees, have her lay a foundation for the store's purchasing orders as business records under Rule 803(6). Then have her read the grand jury transcript to the jury pursuant to Rule 803(5). Correct. This is a problem with double hearsay—the grand jury testimony is one level of hearsay, and the purchase orders are another level of hearsay. Thus, the prosecutor must first use Rule 803(5) in order to allow the witness to read the grand jury transcript as a recorded recollection. Then, the prosecutor must establish that the hearsay within the grand jury transcript (the value of the items, as asserted by the purchase orders) falls under the business records exception to the hearsay rule.

Raymond was on trial for vehicular manslaughter. The prosecutor argued that Raymond drove through a stop sign and struck a pedestrian, killing her instantly. Raymond acknowledges that he drove through the intersection and struck and killed the pedestrian, but he argues that the stop sign was covered up by an overgrown tree and so no reasonable person would have been able to see the sign. To prove his case, his attorney hires an investigator to go to the intersection six months after the accident and take photographs of the stop sign which show that it is blocked by tree branches. Raymond is present at the time and watches the investigator take the photos. At trial, Raymond wishes to admit the photographs as evidence. What procedure should he follow?

Call Raymond to the stand, show him the photos, and have him confirm that these photos are a fair and accurate representation of how the intersection and the stop sign appeared at the time of the accident. Correct. This lays a proper foundation for the photos, since Raymond is providing sworn testimony that this is how the intersection looked at the time the accident occurred. This gives the photographs a very high probative value in the case.

The police pulled Maxwell over for speeding and got his consent to search his car. Inside his car, they found sixteen small baggies of crack cocaine. Maxwell was arrested and charged with possession of cocaine with intent to sell. Maxwell has made no statement at the time of his arrest regarding his intent, but during plea negotiations he admitted that he intended to sell the drugs, and he was willing to plead guilty and get a reduced sentence in exchange for giving information about his supplier. However, Maxwell ultimately backed out of his plea deal and the case went to trial. At trial, the prosecutor needs to prove that Maxwell intended to sell the cocaine instead of merely possessing it for his own personal use. What is the best way for the prosecutor to try to prove this fact?

Call a police officer with significant experience in the field of narcotics and qualify her as an expert. Then ask her if sixteen baggies is consistent with personal use. Correct. A police officer could have this level of expertise from working in the field of narcotics. Although Rule 704(b) precludes an expert from testifying about the mental state of a defendant, most courts allow narcotics officers to testify as to whether a certain amount of narcotics is consistent with personal use.

Denise and Linda were involved in an automobile accident, and Linda's car was destroyed. Denise was uninsured. At the scene of the accident, Denise apologized to Linda and wrote her a note which said: "I hereby state that this accident was my fault. I will pay whatever reasonable damages are due to Linda." A few days later, Linda sought payment from Denise. Denise refused to pay anything, saying she now thought the accident was Linda's fault. Linda sued Denise. At trial, Linda attempted to admit the note that Denise wrote at the scene in which Denise accepted responsibility for the accident. Denise denies ever having written the note. How can Linda prove that Denise wrote the note?

Call an expert in handwriting analysis, who can compare a known sample of Denise's handwriting to the note and testify that the handwriting is the same in each. Call a witness who is familiar with Denise's handwriting, show the witness the note, and have her identify the handwriting on the note as coming from Denise. Linda can take the stand herself, testify that she saw Denise write the note, and that she kept the note with her at all times since then, and that it is in the same condition now as it was when Denise first wrote it. Correct. Under Rule 901(b)(3), an expert can testify that the handwriting sample matches the note. Under Rule 901(b)(2), a non-expert who is familiar with the handwriting can also testify that this is Denise's handwriting, as long as the non-expert did not become familiar with Denise's handwriting for the purposes of this litigation. And under Rule 901(b)(1), Linda can testify that this note is "what it is claimed to be"—a note that she observed Linda write.

Which of the following pieces of evidence is irrelevant under Rule 401?

During a political rally protesting police treatment of racial minorities, Donald allegedly broke a window of the police headquarters. Donald is now being prosecuted for criminal damaging. The elements of criminal damaging are: recklessly or knowingly causing damage to state property without permission or authority. Donald is Black. In Donald's defense, he wishes to present statistical evidence that the police use excessive force when arresting Black suspects at a rate three times more often than when they arrest white suspects. Correct. The only relevant issues in the case are whether Donald caused the damage, whether he did so knowingly or recklessly, and whether he had permission. Whether he believed his actions were justified in order to make a political point is irrelevant to determining his guilt.

Which of the following is not permissible?

During the jury instructions at the end of a trial, the judge tells the jury his opinion about the credibility of each of the witnesses in the case. Correct. the judge's action is not permissible conduct. Rule 605 prevents the presiding judge from "testifying" at the trial, and a comment on the credibility of a witness counts as testifying.

Sam was riding his motorcycle when he was struck by George, who was driving a car. Sam sued both George and Tonda, the car manufacturer. Tonda reached a settlement with Sam, admitting that the brakes on the car were faulty as part of the settlement and paying Sam $100,000 in damages. George and Sam also had settlement negotiations, but they failed to reach an agreement, and the case went to trial. During the settlement negotiations, George admitted that he had drunk two glasses of wine that night before driving, but he claimed that he was still sober enough to see Sam and brake in plenty of time, and the car would have stopped if the brakes had been working properly. At trial, George took the stand in his own defense and testified that he had not had anything to drink that night and that the accident was due to the faulty brakes. At this point in the trial, which of the following is admissible?

Evidence that Tonda admitted to Sam that the brakes were faulty, offered by George in order to prove that the faulty brakes, and not George's negligence, caused the accident. George's statement from settlement negotiations that he drank two glasses of wine before driving, offered by Sam to impeach George's in-court testimony. George's statement from settlement negotiations that he drank two glasses of wine, offered by Sam in order to prove that George's driving was impaired. None of the above. Correct. Rule 408 bars all evidence of statements made during settlement negotiations if offered to prove liability or to impeach the party who said them with a prior inconsistent statement. Rule 408 also bars the admission of completed settlements "on behalf of any party." Thus, any statement made by Tonda during settlement negotiations is also inadmissible at trial to prove that Tonda was liable.

After a plaintiff lost her lawsuit, a juror contacted the plaintiff's attorney with some disturbing revelations about the jury's deliberations. The juror told the plaintiff's attorney that Juror #10, an elderly man, was asleep during most of the deliberations, and that when it came time to vote on the verdict, he seemed confused and only raised his hand after he looked around and saw everyone else raising their hand. The juror also said that during deliberations Juror #5 repeatedly quoted articles from the local newspaper about the case that were sympathetic to the defendant, even though the judge had told the jurors not to read or discuss any news reports. After hearing the juror's report, the plaintiff's attorney wrote up a motion seeking a mistrial and had the juror sign an affidavit in support of the motion and filed it with the court. How should the trial judge rule on the motion?

Grant the motion, but in deciding whether to grant a mistrial, only consider the information about Juror #5's quotes from the newspaper. The trial court is not allowed to consider evidence about Juror #10's alleged sleepiness and confusion. Correct. Rule 606(b) prevents a trial judge from considering any "incident that occurred during the jury's deliberations," but Rule 606(b)(2) states that there is an exception for "any extraneous prejudicial information that was improperly brought to the jury's attention." A newspaper report would count as "extraneous" information.

Greg was in prison, serving a lifetime sentence for rape. While in prison, he got into a fight with another inmate. Both inmates pulled out homemade knives, and Greg was stabbed in the chest. He fell to the floor and was bleeding profusely. As the guards rushed in to help, Greg grabbed one of them and said: "Listen. I don't have much time left. You should know that I killed a person three years before I was convicted of rape. The person's name was Charles Grundy, and he lived in Albany, New York." Greg then fell unconscious. The prison staff tended to his wound, but he died two days later. The guard investigated the case of Charles Grundy, and found out that indeed he had been murdered a few years ago. He also learned that a person named Kyle Worthington had been convicted of the murder, though Worthington always claimed he was innocent and that someone else committed the murder. A few months later, Worthington's conviction was overturned on appeal, and he was re-tried. The guard was called to testify at Worthington's new trial to tell the jury that Greg had confessed to the crime. The prosecutor argued that the confession was hearsay. Is Greg's confession a dying declaration?

Greg's confession is not admissible as a dying declaration because the content of the statement concerns a separate crime. Correct. A dying declaration must be about the cause or circumstances of the declarant's imminent demise. Greg's statement is about someone else's death.

Which of the following is permissible?

In a jury trial, the judge asks questions of one of the witnesses after the direct and cross-examination. Correct. Rule 614(b) allows a judge to ask questions of a witness.

Jane Young, a six-year-old girl, was given antibiotics for a minor infection. Unfortunately, she was allergic to the antibiotics and she became very ill, spending over two weeks in the hospital. After Jane got ill, the doctor met with his two nurses and told them that they needed to conduct a basic allergy test on every new patient with an infection before the doctor could determine which antibiotics should be prescribed. Jane's parents sued the doctor who prescribed the antibiotics, alleging that he should have known that Jane was allergic to this specific strain of antibiotics and thus should not have prescribed them. The Young family had health insurance, so they did not have to pay any expenses aside from a $100 deductible for the hospital stay. However, they sued the doctor seeking medical expenses for the hospital stay (which would ultimately go to reimburse the insurance company, except for the $100 deductible) and for mental suffering (which the Young family would keep for themselves). At trial, Daniel Young, Jane's father, testified about his mental suffering, saying that there were many days when he did not know whether his daughter would live or die, and that he had to seek therapy in order to recover from the trauma. Daniel also testified that he suffered further mental anguish because "Hospitals are so expensive, and I would lie awake at night wondering how we were going to pay for her care and whether we would be able to afford the treatment she needed." Which of the following evidence is admissible?

In his case-in-chief, the doctor defendant seeks to admit evidence that the Young family was fully insured for the hospital stay and only paid a $100 deductible. Correct. Although Rule 411 precludes evidence of insurance to prove negligence or wrongful conduct, the evidence of insurance in this context is being offered to prove that the Young family did not suffer additional mental trauma because they were worried that they would be unable to pay the medical expenses. In other words, Daniel Young "opened the door" to the evidence of the insurance being admissible when she claimed that she was worried about paying the medical expenses.

Gary and Thomas were arrested for conspiring to kidnap a six-year-old child in their neighborhood. A few hours after the arrest, Gary was taken to the prosecutor's office and the prosecutor read him his Miranda rights. The prosecutor talked with Gary for over an hour, and Gary ultimately agreed to plead guilty and testify against Thomas in exchange for a reduction in the charges against him. Gary later changed his mind and decided not to plead guilty. Thomas and Gary were tried separately for the conspiracy. Gary testified in his own trial, and Thomas also called him as a witness in Thomas' trial. When, if ever, can the prosecutor use the statements that Gary made to her after his arrest?

In the trial against Thomas, in order to impeach Gary if his testimony is inconsistent with his statements. Correct. The statements made by Gary in the prosecutor's office were statements made during plea discussions, and therefore the prosecutor would be barred under Rule 410 from using them against Gary for any purpose. However, Rule 410 does not bar the prosecutor from using them against another defendant, only against "the defendant who made the plea or participated in the plea discussions."

Melinda is suing her former employer, Dollar Bank, claiming that her supervisor Reed sexually harassed her for months while she was working there. In its defense, Dollar Bank will call Reed to the stand. He will testify that he has supervised twenty different women over the course of the past three years, and that none of them have ever accused him of any inappropriate sexual behavior. This evidence is:

Inadmissible because of the bar on character evidence. Correct. In a civil case, neither side can admit character evidence to prove propensity under Rule 404(a). The only relevance of this evidence is that it tends to show that Reed has a propensity to not harass his female employees, and therefore he is less likely to have harassed Melinda. As such, this testimony is character evidence and barred under Rule 404(a).

Granger Machines manufactures farm equipment and vehicles. John Kerling, a farmer, owned a harvester that had been manufactured by Granger. On January 5th, the harvester malfunctioned while Kerling was using it, and Kerling's arm was cut off. One week later, on January 12th, Granger sent out a recall letter telling all owners of the harvester to take them into the local Granger dealer, to add on a safety shunt that would prevent such accidents in the future. Daniel Sampson was another farmer who owned a Granger harvester. On January 13th, Sampson's hand was cut off by the harvester. Although the recall letter had been sent out at that point, Sampson had not yet received it. On January 15th, Kerling and Sampson each separately sued Granger Machines, alleging that the harvester had a design defect. In their respective trials, Kerling and Sampson each attempted to admit the recall letter sent out by Granger as evidence that the harvester was defective. Is the recall letter admissible for this purpose?

It is admissible in Sampson's case against Granger, but not Kerling's case against Granger. Correct. The letter is a subsequent remedial measure with regard to Kerling, because it was an action taken by the defendant to make the harvester safer, and it was issued subsequent to the injury to Kerling. Thus, under Rule 407 it is inadmissible in Kerling's case if offered to prove the existence of a design defect. However, the letter was issued before the incident which led to Sampson's injury, so it is not a subsequent remedial measure with regard to Sampson and therefore will be admissible.

Linda drank too much alcohol at a college fraternity party and passed out on a bed upstairs. While she was unconscious, someone came into the room and sexually assaulted her, although the perpetrator left no DNA evidence. After she woke up, Linda discovered what had happened and reported the incident to the police. The police conducted an investigation and ultimately arrested Daniel, one of the members of the fraternity that hosted the party, and charged Daniel with sexual assault. Daniel maintains his innocence, arguing that he never went upstairs at all during the party and never even saw Linda that night. The prosecutor wishes to call (1) George, one of Daniel's fraternity brothers, who will testify that he knows Daniel well and that Daniel has a reputation for taking advantage of women who had been drinking and making sexual advances towards women against their will; and (2) Jane, who met Daniel when she was drinking at a fraternity party six months ago, and who will testify that Daniel had fondled her breasts even though she repeatedly told him to stop and continuously tried to push him away. Should George and/or Jane be allowed to testify?

Jane should be allowed to testify about Daniel's prior sexual assault under Rule 413 (subject to Rule 403), but George's testimony is barred by Rule 404(a)'s ban on propensity evidence. Correct. Although Rule 404(a) generally bars evidence of a defendant's propensity if offered to prove that he acted in accordance with that propensity, Rule 413 creates an exception to that rule if the prosecutor offers evidence of specific acts of sexual assault in a case where the defendant is accused of sexual assault. Rule 413 does not allow a prosecutor to admit reputation or opinion evidence, so George's reputation evidence about Daniel is still barred by Rule 404(a).

Which of the following does not violate the rules of evidence?

Seven days after the verdict is handed down, one of the jurors testifies in a post-conviction hearing that one of her fellow jurors made racially insensitive comments about the defendant, arguing to the other jurors that he believes that members of the defendant's ethnic group are prone to violent actions. Correct. Rule 606(b) forbids a juror from testifying about deliberations, but the Supreme Court in Pena-Rodriguez v. Colorado held that the Sixth Amendment requires the trial judge to hear any evidence of racial bias in jury deliberations.

Jenny and Harold met in a bar, and at the end of the night Harold offered to drive Jenny home. Instead of driving her home, he drove her to a friend's apartment and said that if she came inside he could get some marijuana from his friend and they could smoke it together. Jenny agreed. When the two of them got inside the apartment, however, their stories differ. Harold claims that his friend was not around, but that Jenny and he did smoke marijuana together and then had consensual intercourse. Jenny claims that there was no marijuana, and Harold forcibly raped her. Jenny went to the police with her version of the story the next day, and Harold was arrested and charged with rape. At trial, the prosecutor seeks to call Kathryn, who will testify that Harold raped her in the bathroom of a friend's house when they were at a party together. Kathryn never contacted the police about the alleged rape, so Harold was never charged. Harold seeks to call Stanley, who will testify that two weeks before Harold allegedly raped Jenny, Stanley and Jenny smoked marijuana together and then had sex. Should Kathryn and/or Stanley be allowed to testify?

Kathryn's testimony regarding Harold's alleged prior rape is admissible, subject to Rule 403, because it is evidence of specific act of sexual assault offered in a sexual assault prosecution. Stanley's testimony regarding his alleged sexual intercourse with Jenny is inadmissible because it is offered to prove propensity and violates the rape shield law. Correct. Rule 413 is an exception to Rule 404(a) which allows a prosecutor to admit evidence of prior sexual assaults against a criminal defendant charged with sexual assault. Rule 404(a) and the rape shield law, Rule 412, will bar the defendant from offering evidence of the victim's prior sexual conduct in this case.

Garrett's dog bit Kyle in the leg. Kyle sued Garrett, seeking monetary damages. Under the applicable law, every dog owner is strictly liable for any injuries caused by his dog. At trial, Kyle offers evidence that Garrett's dog bit three other people in the year before biting Kyle. In his defense, Garrett offers evidence that Kyle kicked the dog just before the dog bit Kyle. Is either piece of evidence admissible?

No, because both pieces of evidence are irrelevant. Correct. Since the law is strict liability, it is legally irrelevant whether the dog was known to be dangerous or whether Kyle provoked the dog.

Sandra and Bill made an oral contract in which Bill agreed to buy 12 different rare baseball cards from Sandra. The day after the two of them allegedly made the deal, Bill wrote down from memory the 12 different cards he had agreed to purchase as well as the purchase price for each card. The total purchase price according to his document was $5,280. The next day, Sandra delivered the cards and demanded $8,560. Bill protested, saying that was more than the price they had agreed on. Ultimately the two of them could not agree, and Sandra sued Bill for breach of contract. At trial, Bill testified about the deal he made with Sandra, but he could not remember exactly how much he had agreed to pay for each card. His attorney asked if he wrote down the prices for the cards. Bill said he did, and he confirmed that at the time he wrote down the prices, the prices were fresh in his memory and the record accurately reflected his memory at the time. His attorney then gave him the document he made the day after the deal was made, and (over Sandra's objection) Bill read it out to the jury. Bill's attorney then admitted the document into evidence (again over Sandra's objection). While the jury was deliberating, they asked to see the document, and (again over Sandra's objection) the document was sent back into the jury room for the jury to examine. Did the trial judge make the proper rulings?

No. Although it was correct to allow Bill to read the document to the jury, the document should not have been admitted into evidence or given to the jury. Correct. Bill was allowed to read the document to the jury under Rule 803(5) as a recorded recollection, but the document was not admitted into evidence and so cannot go back into the jury room.

Sam is on trial for raping Rosemary. Sam wishes to call Frank as one of his witnesses. Frank knows Rosemary well and used to date her. He will testify that he is familiar with Rosemary's reputation and that she is known as a woman who frequently engages in casual sexual intercourse. He will also testify that when he dated her, she was "sexually aggressive" and frequently initiated sex with him. Is Frank's testimony admissible?

No. Rule 412 precludes the defendant in a sexual assault case from offering any evidence about a victim's sexual predisposition. Correct. This type of evidence is covered by Rule 412, the rape shield law, and none of the exceptions to Rule 412 apply, so the defendant is precluded from offering this type of evidence.

Norris is suing Stanley for breach of contract. Norris claims that Stanley sent him a letter, which Stanley signed, in which Stanley agreed to sell Norris a vintage comic book for $500. Stanley claims he never wrote any such letter and never accepted Norris' offer. Which of the following is a correct statement regarding the extent to which Norris can testify about the letter?

Norris cannot testify to the statement that Stanley made in the letter; he must have a photocopy of the letter or the original letter, and have it authenticated by someone who knows Stanley's signature. Correct unless Norris shows that the original was lost or destroyed. Rule 1002, the "best evidence rule," states that if a writing is admitted to prove its content, the writing itself must be admitted. Rule 1003 states that a "duplicate" of the writing is admissible to the same extent as the original.

Officer Farnsworth, Officer Young, and Officer Deason responded to a 911 call at a bar and found a man named Quincy throwing beer mugs and screaming at the other patrons. One of the beer mugs had hit another man in the head, and the man was bleeding severely. Officer Farnsworth began tending to the injured man, Officer Young approached Quincy, and Officer Deason stayed back to provide cover for Officer Young. Quincy had a broken beer bottle in his hand and was threatening Officer Young with it. Within a few seconds a struggle ensued, in which Officer Young wrestled Quincy to the ground and began kicking him in the head to subdue him. Quincy was ultimately arrested, and he suffered permanent brain damage as a result of the kicks. Quincy sued the police department, arguing that Officer Young used excessive force in apprehending him. The defendant police department called both Officer Farnsworth and Officer Deason as witnesses. Officer Deason testified that she observed the entire encounter, and in her opinion Officer Young used necessary force when he kicked Quincy, because Quincy was attempting to stand up and stab Officer Young with the beer bottle and the kicks were the only thing keeping him down. Officer Farnsworth testified that he did not see the encounter because he was tending to the wounded individual, but in response to a hypothetical question from the attorney, he testified that if Quincy were still trying to stand up and had a weapon in his hand, it would be necessary for Officer Young to kick him to keep him down. Neither Officer Farnsworth nor Officer Deason were qualified as experts by the court; they offered their testimony as lay opinions. Was their testimony admissible?

Officer Deason's testimony was admissible, but Officer Farnsworth's was not. Correct. Officer Deason's testimony was rationally based on her perception, was helpful to the jury, and was not based on scientific or expert knowledge. She was using her personal experience as a police officer in order to come to the conclusion that the kicks were necessary, and basing an opinion on personal knowledge does not make the opinion an expert opinion. Thus, her opinion is admissible under Rule 701. However, Officer Farnsworth did not personally observe the kicking, and so his opinion in response to a hypothetical question is an expert opinion and not admissible as a lay opinion.

Sam is suing Super-Mart for employment discrimination, arguing that they refused to promote him because of his race. In his case-in-chief, he offers e-mails written by his supervisor at Super-Mart in which she uses a derogatory racial epithet to describe Sam. Super-Mart objects to the e-mail as improperly authenticated, and Sam's attorney does not respond to the objection. The e-mails are precluded. The jury ultimately finds Super-Mart not liable. On appeal, Sam argues that the e-mails should have been admitted, and he presents in his brief sufficient foundation to authenticate them. However, he never presented this foundation at trial. What should the appellate court do?

Overrule the lower court's ruling and remand for a new trial, with an order that the trial court reconsider the authentication issue in light of the evidence Sam's attorney has presented, but only if the preclusion of the e-mails affected Sam's substantial right and it was plain error for the judge to preclude them. Correct. Rule 103(a)(2) states that a party "may claim error" in an evidentiary ruling excluding evidence only if the party informed the court of its substance by an offer of proof. Sam needed to give the trial judge all the information she needed in order to make a proper ruling at the time of the objection. However, Rule 103(e) allows the appellate court to consider the issue if it was a plain error and the error affected the party's substantial right.

Morris received a flyer in the mail from Able Home Improvement, offering a free estimate for evaluating his home's insulation needs. Morris called Able and asked Able to come out and give them an estimate. Able employees came to Morris' home and spent three hours in his attic inspecting his insulation. When they were done, they provided him with an estimate for new insulation and also a bill for $200 for the estimate. Morris protested, saying that Able's flyer had promised a free estimate. Unfortunately, Morris had thrown the flyer away the day after he called Able to come out, and his garbage had been taken away to the dump. Able demanded the money and ultimately sued Morris for non-payment. At trial, Morris sought to testify about Able's flyer which promised a free estimate. Able objected, arguing that the best evidence rule precluded the testimony. How should the judge rule on the objection?

Overrule the objection, because the original has been lost. Correct. Rule 1004(a) states that the contents of a writing can be proven through other means (such as oral testimony) if the original has been lost, and not by the proponent in bad faith. Even though Morris (the proponent) was the person who lost the original, he did not lose it in bad faith, since he threw it away days before the dispute arose.

The First Union Bank was robbed on September 1. The robbers fled in a car that was seen driving into an area of the city known as the Downs. Officers found the car, abandoned, two hours later in the Downs. The detective in charge of the case reviewed the footage from the bank's security cameras and found one second of film in which one of the perpetrator's faces was uncovered and visible. The detective made a still photo from this one second of footage and then gave it to dozens of officers, who went door to door through the Downs, telling people that they were investigating a crime and asking them if they recognized the photo. Officer Brown was carrying out this assignment when he knocked on the door of Beverly Grogan. He showed her the photo and she gasped, putting her hand to her mouth and saying: "My God, that's my husband!" She then started crying. Officer Brown reported this to the detective, and her husband was ultimately arrested for robbing the bank. At trial, the prosecutor called Officer Brown to the stand and asked him what Beverly Grogan said when he showed her the photo of the bank robber in order to prove that the defendant was the person who robbed the bank. The defendant objects. What is the proper ruling on defendant's objection?

Overrule the objection, because the statement is an excited utterance. Correct. At the time the witness made the statement, the declarant was under the emotional stress of seeing her husband in a photo being held by a police officer—she seemed surprised, she exclaimed, and she began crying. Thus, it is an excited utterance under Rule 803(2).

Greg rented a stand-alone unit from Store-Your-Stuff storage facility in Florida, and he put six boxes containing his rare book collection inside the unit. Two weeks later, a severe rainstorm hit the area, bringing over 3 inches of rain. Because the roofs of the storage units had not been replaced for over twenty years, they could not keep out such a large quantity of rain, and water leaked into Greg's unit, destroying his books. Greg is now suing Store-Your-Stuff, arguing that the facility was negligent in not replacing the roof or taking any other action to prevent leaks. After some investigation, Greg learned that one week after the storm, Store-Your-Stuff installed tarps over all of its units. At trial during his case-in-chief, Greg wants to admit the fact that Store-Your-Stuff put tarps on the roofs of the storage units. Store-Your-Stuff has not yet contested that attaching a tarp to every roof would not be feasible, though Greg suspects that they might do so when they present their defense. How should the court rule on the evidence of the tarps?

Preclude all evidence of the tarp installation in Greg's case-in-chief. Correct. Store-Your-Stuff's installation of the tarps was a subsequent remedial measure, since it was an action taken by the defendant after the incident which was meant to prevent a similar incident from occurring again. As such, it is barred by Rule 407 if offered to prove negligence. Since this is being offered in Greg's case-in-chief, Greg cannot argue that he is offering the evidence to prove ownership, control, or feasibility, because Store-Your-Stuff has not yet presented its case.

Chrome Motor Company developed a new type of SUV called the Growler. Unfortunately, within two weeks after the Growler was out on the market, three different owners reported problems with the accelerator pedal. In all three cases, the driver reported that the accelerator pedal stuck, causing the car to accelerate uncontrollably, resulting in an accident that caused injury and property damages. All three owners sued Chrome, alleging that the Growler had an unsafe design that caused the accelerator to stick. Chrome settled with the first plaintiff for $50,000 and the second plaintiff for $100,000. However, the third plaintiff rejected all settlement offers and took the case to trial. At trial, the attorney for the third plaintiff sought to admit the fact that Chrome settled the other two cases, as well as the amount of money Chrome paid for the settlement. The attorney argued that these settlements proved that Chrome acknowledged the design was faulty. Chrome objected to this evidence. Should the judge admit or preclude the evidence of the settlements?

Preclude any evidence from either settlement. Correct. Rule 408 precludes any evidence of a settlement in order to prove liability, and there is no other reason that these settlements are relevant. Rule 408 applies to completed settlements as well as settlement offers.

Police received an anonymous tip that cocaine was being sold out of an apartment. Officer Jordan went to the apartment undercover to attempt to purchase cocaine. Two other undercover police officers waited outside the apartment building and saw Officer Jordan enter. Officer Jordan stayed inside for five minutes, and then the officers saw him come back outside. The officers meant to join him, but he gave a signal that indicated he believed he was being followed. Instead, the officers met Officer Jordan back at the precinct two hours later. There Officer Jordan told them that he climbed two flights of stairs, knocked on the door of the designated apartment, and bought cocaine from a man with dark curly hair wearing a red leather jacket. The officers immediately returned to the apartment building to make the arrest, and as they climbed the flights of stairs, they saw a man with dark curly hair wearing a red leather jacket walking down the stairs. They arrested the man, whose name was Edward Heath. There were no narcotics found on Heath, and nothing connecting him to the apartment or the drug sale other than the statements of Officer Jordan. Heath was charged with sale of narcotics. Unfortunately, Officer Jordan was killed in a car crash one week before Heath went on trial. One of the officers who talked to Office Jordan at the precinct attempted to testify as to the description Officer Jordan provided about the person who sold him drugs. Heath objected to the testimony as hearsay. What should the judge do?

Preclude the evidence as hearsay. Correct. The evidence is an out of court statement offered to prove the truth of the matter asserted, and there is no hearsay exception which will allow it in.

Harold is suing Avery, claiming that Avery ran a red light in her car and struck him as he crossed the street in a crosswalk. Avery claims that the light was green when she entered the intersection and that Harold was crossing the street against the light. Harold wants to call Ronald, Avery's co-worker. If allowed to testify, Ronald will say that he has ridden with Avery hundreds of times and that Avery has a "habit of being a reckless driver." Ronald will support this claim by testifying that Avery routinely runs red lights, drives at speeds significantly over the speed limit, and is inattentive to the road. Harold objects to this testimony. Should the judge admit or preclude Ronald's testimony?

Preclude the evidence as improper propensity evidence. Correct. Although Ronald will testify using the word "habit," this testimony is not actually habit testimony as defined by Rule 406. Habit is a regular and specific response to a specific type of situation. This testimony is not specific enough to be habit—it is purely propensity evidence and thus inadmissible in a civil case under Rule 404.

Woodlands Construction built a building for Ace Insurance Company. There was a dispute about the final cost of the project: Ace was contractually obligated to pay the full cost of all the building supplies, but Ace claimed that Woodlands was exaggerating the cost of the building supplies in its final invoice. Woodlands sent Ace four letters over the course of a month, demanding the money that it claimed was still owed and detailing the amount that it paid for the each of the supplies, but Ace refused to pay the disputed amount. Finally Woodlands sued Ace. At trial, Woodlands sought to introduce the letters it sent to Ace which described the amount it paid for each of the building supplies. Ace objected to these letters as hearsay. Should the court admit or preclude the letters?

Preclude the letters as hearsay. Correct. The letters are out-of-court statements offered to prove the cost of the building supplies. The letters fail the requirements of Rule 803(6)--they are not records that are kept in the regular course of business, and it is not a regular practice to make this kind of record. Also, the circumstances indicate a lack of trustworthiness. Rule 803(5) only applies when a witness is on the stand and has forgotten certain facts, and so uses his or her own record to get the information to the jury. There is no evidence that such a foundation has been laid here. And Woodlands is offering its own letters; thus these are not statements made by a party opponent, as required by Rule 801(d)(2).

George was loading a truck at work with boxes when the forklift operator accidentally hit him with the forklift, crushing his torso and breaking six ribs. George was rushed to the hospital and prepped for surgery to repair damage to his internal organs. As he was waiting to go into surgery, the Vice-President of the company he worked for came to him and said: "George, this was a terrible thing to happen. I want to assure you that the company will pay for this surgery and any rehabilitation that you need." George is now suing his company for negligent training and supervising of its employees, and he wants to testify as to the Vice-President's statement. Which rule of evidence would be the BEST one for the company to cite when it objects to this evidence?

Rule 409, which bars any offer to pay medical expenses. Correct. Rule 409 bars evidence of any offer to pay medical expenses if offered to prove liability, regardless of whether there was a disputed claim at the time of the offer.

After reviewing Samantha's tax returns for the past three years, the Internal Revenue Service ("IRS") contacted Samantha and asked her to come to the local IRS office to explain some irregularities. The IRS agent suspected that Samantha had intentionally understated her income by over $100,000 and was hoping that he could elicit some incriminating statements from her during the meeting. At the meeting, the IRS agent presented Samantha with the evidence of the discrepancies between her actual income and the income she reported on her taxes, and he told her that she faced significant civil penalties. Samantha admitted that she made a mistake in calculating her taxes, but claimed it was merely an oversight. She offered to pay the minimum fine in order to avoid additional penalties. The IRS agent refused the offer, and the meeting ended. The IRS turned the case file over to the United States Attorney's office, and Samantha was ultimately criminally charged with income tax evasion. At trial, the government sought to admit her statements in which she confessed to understating her income and offered to pay the minimum fine. Samantha objects. Are Samantha's statements admissible?

Samantha's offer to pay the minimum civil penalty is inadmissible, but her statement admitting to understating her income is admissible. Correct. Rule 408 applies in this case, because there was a disputed claim and Samantha was conducting settlement negotiations. However, if a defendant made statements when negotiating a claim by a public office exercising its enforcement authority, those statements are admissible in a subsequent criminal trial under Rule 408(a)(2). The offer to settle itself is still inadmissible under Rule 408(a)(1).

Louis was stopped on the street pursuant to a legal Terry stop and the police officers frisked him and recovered a handgun. The officers ran his records and found that three years ago Louis had been convicted of burglary, a felony. Louis was charged with knowingly possessing a firearm while being a convicted felon. The elements of the crime are that the defendant (1) knowingly possessed a firearm; and (2) the defendant is a convicted felon. At trial, Louis admitted that he had been convicted of burglary three years ago, but he testified that he did not know that burglary was a felony. He will testify that he only received a six month sentence, and that he honestly believed that a crime was not a felony unless the defendant served a year or more in prison. The prosecutor objects to Louis' testimony. How should the court rule?

Sustain the objection because Louis' testimony is irrelevant. Correct. The elements of the crime are that the defendant (1) knowingly possessed a firearm; and (2) the defendant is a convicted felon. Whether the defendant knows he is a convicted felon is irrelevant; thus Louis' testimony about his lack of knowledge is irrelevant.

On the morning of January 24th, Larry and his wife Ingrid got into an argument. Larry slapped Ingrid in the face and threatened to break her nose if she talked back to him again. Ingrid was very upset and began crying, but she did not call the police. Instead, she went to her job as a nurse and worked a full day at the hospital. At the end of her shift, another nurse noticed that she had a mark on her face and asked her what happened. She shook her head and said: "Larry hit me again. Next time he said he would break my nose. I really don't know what to do." Ingrid's co-worker was shocked at the news and convinced Ingrid to call the police. Larry was arrested and charged with assault and menacing. At trial, the prosecutor called Ingrid to testify about what happened. Ingrid took the stand, but then testified that Larry never hit her or threatened her. The prosecutor then called Ingrid's co-worker to testify about what Ingrid told her at the end of the day. The defense attorney objected to this testimony. Is the testimony of Ingrid's co-worker admissible?

The co-worker's testimony is inadmissible to prove the truth of the matter asserted, but it is admissible to impeach Ingrid. Correct. The statement is hearsay, and there is no exception to admit it. However, it contradicts Ingrid's testimony, so it is admissible to impeach her under Rule 613 as a prior inconsistent statement. Since it is not a collateral statement (it is relevant for a purpose other than to impeach Ingrid), the prosecutor is allowed to use extrinsic evidence (such as the co-worker's testimony) to prove the statement was made.

An oil tanker belonging to Davis Oil ruptured, and the resulting oil spill damaged the coastline of Florida and Georgia. The Environmental Protection Agency ("EPA") sent a team out to investigate the extent of the damage. The EPA team interviewed fishermen and coastal residents, and issued a report. Local businesses and fishermen are suing Davis Oil in a class action suit. They seek to admit parts of the EPA report as evidence against Davis. Which parts of the report (if any) are admissible?

The conclusion of the report, which states that "the damage to the local economy exceeds $300 million, and Davis Oil did nothing to mitigate this damage during the first three days after the oil spill." Correct. This is admissible as a public record under Rule 803(8)(A)(iii). It sets out factual findings as a result of a legally authorized investigation of a public office.

Defendant Davidson Motors manufactures automobiles. On February 15th of this year, a motor in one of its cars spontaneously caught fire, causing property damage to the surrounding area. The plaintiff is the owner of the car and is now suing Davidson, alleging that the motor was defectively designed. It is undisputed that Davidson issued a recall letter on or about February 15th, ordering all of its customers to take their cars into a dealership for mechanical work that would fix the defect that caused the plaintiff's car to catch fire. There is no dispute that the recall letter, if it was issued after the plaintiff's motor caught fire, would be a subsequent remedial measure under Rule 407 and would therefore be inadmissible in the plaintiff's case if offered to prove liability. However, the plaintiff argues that the defendant issued the recall letter before the plaintiff's car caught fire in response to the defendant's own testing. The defendant argues that it issued the recall letter after the plaintiff's car caught fire in response to the plaintiff's accident. The defendant offers affidavits from its chief safety officer and its marketing director in which the affiants assert that the recall letters were issued on February 16th, after the defendant learned about the plaintiff's car. What is the proper procedure to determine the admissibility of the recall letter?

The decision of whether the recall letter was issued before or after the plaintiff's car caught fire is a question for the judge to decide, and the judge should do so outside the presence of the jury. The judge is allowed to consider the affidavits in making her decision, even though the affidavits are otherwise inadmissible hearsay. Correct. Under Rule 104, all questions of admissibility are decided by the judge and not the jury, and Rule 104 also states that the judge is not bound by the rules of evidence (other than the privilege rules) in making those decisions.

Gail is an eyewitness in a hit-and-run prosecution. When the prosecutor asks her for the license plate of the car that she saw strike the pedestrian, she states that she cannot remember the license plate number. The prosecutor asks if seeing a photo of the license plate would refresh her recollection, and Gail says yes. The prosecutor has a photo of the license plate. What is the proper procedure to refresh the witness' recollection?

The witness should look at the photo and then it should be taken away from her and she should testify about the license plate number from her refreshed recollection. Correct. The proper procedure for refreshing recollection under Rule 612 is to have the witness look at the writing, then take it away and have her testify from her refreshed recollection. Any item that refreshes recollection can count as a "writing." The writing cannot be admitted as evidence by the party who is using to refresh recollection.

Winslow Laboratories is suing Hanover, Inc. for patent infringement. As part of the discovery process, Winslow deposed five of Hanover's employees, including Hanover's Vice-President in charge of Research and Development. Later, the Vice-President also submitted a sworn affidavit relating to the case. At trial, Hanover decided not to call the Vice-President as a witness because Hanover's lawyers were worried that he would not seem credible to the jury. Instead, Hanover sought to admit the Vice-President's deposition testimony and his affidavit. The evidence is being offered for the truth of the matter asserted. Winslow objects to this as hearsay. Should the judge admit or preclude the Vice-President's deposition testimony and affidavit?

The deposition and affidavit are hearsay and should be precluded. Correct. Rule 804(b)(1) provides for an exception for prior testimony which was given under oath with an opportunity for cross-examination, but the rule only applies if the declarant is now unavailable. In this case, the declarant (the Vice-President) is available, but the defendant simply chose not to call him to the stand. There is no other hearsay exception which will allow this testimony.

Police officers lawfully intercepted an e-mail sent from the account [email protected]. The e-mail offered to sell two hundred handguns, and it was sent to an undercover police officer. The prosecutor wants to authenticate this email as belonging to Grace Fell, who is being charged with illegal distribution of handguns. Which (if any) of the following pieces of evidence would likely be sufficient to authenticate the e-mail as having been written by Grace?

The e-mail account [email protected] is registered to Grace Fell and the content of the e-mail contains accurate, personal information about Grace, including her home address and a reference to her dog Butch. Correct. This is sufficient evidence to show that the item is what its proponent claims it to be. Rule 901(b)(4) states that the distinctive characteristics of the item can help to authenticate that item.

Vicky is suing her employer, Mershon Bank, for sexual harassment. She alleges that her immediate supervisor created a hostile work environment by making sexually explicit jokes in her presence and asking about her intimate relationships. In its defense, Mershon seeks to admit evidence that Vicky frequently told sexually explicit jokes at work and that she had a reputation as being sexually promiscuous with her co-workers. Is this evidence admissible?

The evidence about the sexually explicit jokes is probably admissible, but the evidence about her sexual reputation is not. Correct. Under Rule 412(b)(2) , the evidence that Vicky told sexually explicit jokes is probably admissible because its probative value substantially outweighs the danger of harm to any victim or of unfair prejudice to any party, since it tends to show that Vicky would not find that behavior at work offensive. However, evidence of the victim's reputation is inadmissible unless the victim herself places it in controversy, which she did not in this case.

Debra was a fourteen-year-old girl who was being treated for severe depression. During her second session with her psychologist, she told him that her stepfather had sexually abused her, and she described the abuse in detail. This information was critical to determining how to treat Debra's depression. The stepfather was arrested and charged with child molestation. Debra refused to testify at the trial. The psychologist was called to the stand to testify about what Debra told her. The stepfather objects to this testimony as hearsay. How should the court rule on this exception?

The exception for medical treatment or diagnosis applies and the statement is admissible. Correct. Rule 803(4) generally does not apply to statements of fault, but in some cases (as with statements regarding emotional trauma brought on by family members), statements of fault are pertinent to medical treatment or diagnosis.

Agents from the Drug Enforcement Agency got an anonymous tip that someone was selling drugs out of a black car parked outside Happy Times bar. When they arrived at the Happy Times bar, they saw a black car in the parking lot. As they were waiting outside the bar, a woman came out of the bar and opened up the trunk of the car. As soon as she opened the trunk, one of the agents approached her and showed her his badge. When the woman saw the badge, she immediately took off running back into the bar. One of the agents caught her, and another one looked inside the open trunk and found a paper bag with ten small baggies of cocaine inside. The woman was arrested and identified as Phoebe Gates. She was charged with knowingly possessing cocaine. At trial, the DEA agent wanted to testify about the fact that Phoebe ran as soon as she saw the law enforcement badge. Phoebe's attorney objected. Is the fact that Phoebe ran admissible?

The fact that Phoebe ran is admissible because it tends to prove she knew there was cocaine in the trunk. Correct. The prosecutor has to prove the Phoebe had knowledge of the cocaine in order to prove possession, and the running tends to prove knowledge. Phoebe was not trying to assert anything by running, so her running does not constitute assertive conduct.

Rick owned a home near a river on a flood plain, and one year after many days of rain, the river rose and flooded his house, causing severe damage. Rick contacted his insurance company, but the insurance company refused to pay anything for the damage. Rick ultimately sued the insurance company for the damages he suffered. The insurance contract that Rick and the company signed was ambiguous as to whether the policy covered flood damage for houses inside the flood plain. Under the relevant contract law, the oral representations of the parties that were made when signing the contract are relevant when a court is interpreting the meaning of the contractual provisions. Rick testified that the insurance agent never told him that flood damage was not covered. The defendant insurance company called the agent who sold Rick the policy. The agent testified that he honestly did not remember what he said when he sold Rick the policy, since he sold the policy ten years ago. The agent will testify that every time he sells a policy to a homeowner whose home is in the flood plain, he notifies the homeowner that the basic policy does not cover flood damage. He estimates that he has sold hundreds of these policies during his career as an insurance salesman. Rick objects to this testimony. Is the insurance agent's testimony admissible?

The insurance agent's testimony is admissible as habit evidence. Correct. Under Rule 406, evidence of the routine practice of an individual is admissible as habit evidence if it is specific conduct that is routinely done in response to specific situations. Here, the insurance agent testifies that he "always" warns homeowners about the flood plain exemption, and this is very specific conduct.

Debra is on trial for shooting her husband in the back while they were out hunting together. Debra admits that she shot him, but claims that the shooting was accidental; she was shooting at a deer and did not see her husband in front of her. The prosecution wants to admit evidence that one week before the shooting, Debra attempted to poison her husband by placing rat poison in his morning coffee. Her husband did not drink the coffee, but the housekeeper found the undrunk cup and will testify that it "smelled bitter, like the poison I set out for the rats." The housekeeper noticed that the box of rat poison had been moved that morning by someone other than herself. The housekeeper will also testify that Debra always made the coffee for herself and her husband. Should the judge admit the housekeeper's testimony?

The judge should admit the housekeeper's testimony if the judge believes that a reasonable jury could find by a preponderance of the evidence that Debra put rat poison in the coffee. Correct. The housekeeper's testimony is evidence that Debra had the intent to kill her husband, which is relevant, especially given the fact that Debra's defense is that the killing was accidental. When deciding admissibility of a prior bad action under Rule 104(b), a judge is deciding whether there are sufficient facts to determine whether evidence is relevant. The evidence will not be relevant unless a jury is likely to find that the act actually occurred. Thus, the judge must consider not whether she thinks the prior act took place, but whether a reasonable jury could believe a prior act took place.

Levi is charged with raping Irene after the two of them went out on a date. Levi and Irene lived in the same college dormitory at the time. Levi claims he believed the sex was consensual; Irene testifies that she clearly indicated she did not want to engage in sexual intercourse and attempted to push Levi away, but was unable to prevent the act. As part of her case-in-chief, the prosecutor wants to call Kristine, who will testify that Levi sexually assaulted her after a date in high school six years earlier. Kristine reported the crime to the police, but she says the police did not take her seriously and no charges were ever filed. For his part, Levi seeks to call Nancy, who lived in the same dorm as Irene and Levi. Nancy will testify that everybody in the dorm knew Irene's reputation, and that she was someone who would always consent to sex after a date, and would usually pretend to protest because "that made it more fun for her." Should the judge allow the testimony from Kristine and/or Nancy?

The judge should allow Kristine to testify to prove Levi's propensity to engage in violent sexual acts against women (subject to a Rule 403 analysis), but should preclude Nancy's testimony. Correct. If a defendant is accused of sexual assault, Rule 413 makes it possible for the prosecutor to potentially bring in any prior accusation of sexual assault to prove propensity. Rule 412 will bar the evidence of Nancy's reputation; there is no exception to Rule 412 for proving the defendant's "state of mind," and courts have consistently refused to apply Rule 412(b)(1)(C) to include such scenarios.

Michael is on trial for murder. The prosecutor's theory of the case is that Michael stabbed his roommate six times in the chest while his roommate was sleeping. The prosecutor wishes to admit three photographs of the crime scene as it appeared when officers first arrived, showing the victim in the bed, with the stab wounds clearly visible and blood soaking through his sheets and nightclothes. Michael's attorney objects to the photos, arguing that they will unfairly prejudice the jury. Should the judge preclude the photos?

The judge should preclude the evidence if the risk of unfair prejudice substantially outweighs the probative value of the evidence. Correct. This is the correct test under Rule 403.

The government is suing Bloring Finance for discriminatory lending, arguing that Bloring offered lower interest rates on loans to white customers than it did to Black customers, and that the difference in rates was because of the customer's race rather than a legitimate reason, such as income level or credit rating. Under the applicable federal law, if the average interest rate that is offered by a company to a protected class such as a racial minority is more than 1% higher than the interest rates offered to the general population, the company is presumed to have a discriminatory intent. The government proved by a preponderance of the evidence in its case in chief that the interest rate Bloring offered to the general population was 5%, and the interest rate Bloring offered to Blacks (a protected class) was 7%. Under Federal Rule 301, what is the effect of the presumption?

The jury must conclude that Bloring had a discriminatory intent, unless Bloring presents some evidence that it did have a discriminatory intent, in which case the presumption disappears and the jury should conclude that Bloring had a discriminatory intent only if it is convinced by a preponderance of the evidence that Bloring had a discriminatory intent. Correct. This describes a "bubble bursting" or Thayer presumption—if a party proves A is true, then B is proven true unless the opposing party presents some evidence that B is false. Once the opposing party meets this burden of production that B is false, then the presumption disappears and the burden of persuasion returns to what it would have been without the presumption. This is the presumption described in Rule 301 as interpreted by federal case law.

Wilbur was stabbed in the chest outside of a bar late one night. The police responded to the call within minutes, and found Wilbur breathing heavily, crying, and in a lot of pain. Wilbur said: "A man named Hank stuck me with a knife! He is wearing a red sweatshirt and has short black hair. He ran south down the street." At the time of Wilbur's statement, Hank was still at large and, as far as Wilbur and the police knew, still carrying a knife. The police located Hank Johnson hiding in an alley down the street and arrested him. He was wearing a red sweatshirt and had short black hair. At Hank's trial, the prosecutor called Wilbur, but he refused to come to court. Instead, the prosecutor called the police officer who responded to the scene and asked him to testify about what Wilbur told him. Hank objected, arguing that Wilbur's statement was hearsay. Is Wilbur's statement admissible?

The statement is admissible as an excited utterance and it does not violate the Confrontation Clause. Correct. Wilbur was under the excitement of the event at the time he made the statement, so his statement fits the excited utterance hearsay exception under Rule 803(2). And the statement does not violate the Confrontation Clause because it was made during an ongoing emergency and the primary purpose of the police interrogation was not to develop evidence for trial but instead to help the police assist during the emergency. Under Michigan v. Bryant, this is a non-testimonial statement.

Nadine was shot in the chest in her home. She was found by her sister two hours later, barely conscious. Her sister asked her what happened, and she answered in a calm, clear voice: "My boyfriend Chris shot me," and then she lost consciousness. The police located Chris Dawkins, her boyfriend, and arrested him. Nadine is now in a coma. Chris is being prosecuted for attempted murder. The prosecutor calls Nadine's sister to the stand to testify about what Nadine said. Chris objects, arguing that Nadine's statement is inadmissible hearsay. What is the proper ruling?

The statement is inadmissible hearsay. Correct. Nadine's statement was an out-of-court statement, and the prosecutor is using the statement to prove the truth of the matter being asserted. The dying declarations exception (Rule 804(b)(2) almost applies - Nadine almost certainly thought she was going to die, the statement relates to the cause of her death, and she is now unavailable. However, in a criminal case, a dying declaration is only admissible if the crime is a homicide—that is, if the victim actually dies. And the party-opponent statement exemption does not apply, because Nadine is not a party to the case. Thus, the statement is inadmissible hearsay.

Peter is suing Downing Lawn Vehicles. Peter claims that he was using a model 6100 Downing riding mower on his lawn, and when he turned the speed to maximum, one of the front wheels detached from the axle, causing the riding mower to crash into a tree and throwing Peter from the mower. Peter claims that these mowers suffer from a design defect which causes the front wheels to occasionally detach from the axle when the mower is driven at a high speed. As evidence to prove his allegation, Peter wants to admit the testimony of three other individuals who used a model 6100 Downing riding mower the year before Peter had his accident. In each case the front wheel fell off the axle when the owners drove them at maximum speed. All three of these individuals will also testify that they wrote Downing a letter complaining of the incident immediately afterwards, and Peter will seek to admit the letters that they wrote. What is the proper ruling?

The testimony is admissible to prove that this model riding mower had a design defect and the letters are admisible to prove that Downing had notice of the defect. Correct. The testimony is relevant to the problem with the mowers. The fact that other machines of the same model were faulty is evidence that the model itself was improperly designed or manufactured, which tends to prove the fact that Peter's mower was improperly designed or manufactured. And the letters are only being admitted to prove that Downing was put on notice that there was a problem with the mowers, and so there is no hearsay bar to admitting them.

Ronald is a truck driver who works as an independent contractor, hauling freight for small businesses across the country. A recycling center in California contacted Ronald and told him it would pay him a set fee if he transported ten truckloads of aluminum cans from their warehouse near San Francisco to the processing plant near Los Angeles. Ronald transported all ten truckloads, and the recycling center paid him $10,000. Ronald claims that the recycling center had promised to pay him $15,000, and he sued them for the difference. At trial, the president of the recycling center takes the stand and testifies: "When we first contacted Ronald, I told him we would pay him $10,000, to haul ten loads." Ronald objects this testimony as hearsay. Is the president's testimony admissible?

The testimony is not hearsay, because it is not offered for the truth of the matter asserted, but instead to prove that an offer with specific terms was made. Correct. The statement is being offered to prove that the statement was made (since the statement itself is a legally operative fact), not for the truth of the matter asserted. Terms of contracts are not considered hearsay, since legally all that matters is that the declarant made the statement. The president's testimony can always be challenged on cross-examination; that is, just because it is admissible does not mean the jury will accept it as fact.

Gregory was severely injured when the axle of his car detached while he was driving, causing his car to swerve off the side of the road and hit a telephone pole. Gregory broke his spine and now cannot walk and has limited use of his arms. He is suing the automobile company that manufactured the car. In his case-in-chief, Gregory will take the stand and describe the extent of his injuries. He has also made a short video depicting his daily routine in the wake of the accident. The video shows him brushing his teeth, taking a shower, getting his meals, driving to work, and so on. He offers this video as evidence. The defendant car company objects. Is the video admissible?

The video should be admitted as long as Gregory testifies that the video is a fair and accurate representation of his daily routine. Correct. These "day-in-the-life" videos are generally admissible as demonstrative evidence as long as the witness testifies that they fairly and accurately represent how the injury has affected the plaintiff's life. They are relevant to establish damages and they can be helpful to the jury because they can show the jury difficulties that the plaintiff faces in everyday life, which may be hard to describe on the witness stand. As long as they are a fair and accurate representation of reality, they are not unduly prejudicial, and they do not confuse or mislead the jury.

Ross was at work using a power saw that had been manufactured by Cutter Tools. Ross held the trigger of the saw steady with his left hand and used his right hand to guide the wood through the rotating blade. When the wood was cut, Ross leaned down to pick up the cut piece of wood and took his left hand off the trigger of the saw. However, the saw blade did not stop rotating, and as Ross leaned over to pick up the wood, his right hand came into contact with the saw blade and was severed from his arm. Ross sued Cutter Tools, alleging that their design was unreasonably dangerous because the saw blade did not immediately stop rotating when the user took his hand off the trigger. Which of the following is MOST likely to be admissible?

Two weeks after the incident, the same model of the Cutter Tools power saw severed the finger of a woman using it in her home when the blade kept rotating after she took her finger off the trigger. Correct. Because this was the same model saw, and because the accident happened in a very similar way, this evidence is relevant to proving the saw had a defect. It is irrelevant that the accident happened after the incident with Ross, because the evidence still tends to prove that the design itself was defective.

Benjamin was a confidential informant working for the police. He wore a recording device to a meeting with Karl, a city official, posing as a developer that needed a variance on a zoning regulation. During the meeting Karl said that he would be willing to give Benjamin the variance in exchange for a cash bribe of $100,000. Benjamin took the recording device back to the police and played it for them. The police then made a transcript of the interview. Karl was arrested for soliciting bribes. Benjamin was called to testify about his meeting with Karl. Can Benjamin tell the jury what Karl said during the meeting?

Yes, Benjamin can testify as to the contents of the conversation between himself and Karl. Correct. If the witness (in this case Benjamin) has first-hand knowledge of certain facts (in this case the conversation between himself and Karl), he can testify to these facts even if they have been recorded in a "writing." The best evidence rule does not prohibit a witness from testifying to facts that he knows firsthand just because they have been recorded. And Karl is the defendant in the case and so all of his statements are admissible if offered by the prosecutor as a party-opponent statement.

A police officer pulled over Sam's car after seeing it swerve numerous times into the oncoming lane of traffic. The officer ordered Sam out of the car and told him to take a Breathalyzer test. Sam refused. Based on Sam's erratic driving and Sam's refusal to take the test, the officer arrested Sam for driving while under the influence of alcohol. A person is guilty of this crime if his ability to drive a car is impaired because of consumption of alcohol. At trial, the prosecutor wants the officer to testify that Sam refused to take the Breathalyzer test. Is this evidence relevant?

Yes, because his refusal to take the test makes it more probable that he was in fact intoxicated. Correct. Although there are other reasons why he may have refused to take the test, the legal standard for relevance is very low—the proffered evidence only needs to have some tendency to make a fact in consequence more or less probable. Since refusing to take the Breathalyzer may be probative of consciousness of guilt, the evidence is relevant.

Oswald is suing his former employer, Overton Power, in federal court. Oswald claims that after he worked at Overton's nuclear power plant for eight years, he developed cataracts in his eyes as a result of the high radiation levels at the plant. Overton states that there is no evidence that Oswald's cataracts were caused by high levels of radiation. In his case-in-chief, Oswald seeks to call Dr. Conway, an ophthalmologist who examined Oswald's eyes. Dr. Conway will testify that he was able to determine that Oswald's cataracts were caused by radiation after an intensive visual examination. Oswald objects, arguing that there is no way to determine whether cataracts are caused by radiation without a biopsy, in which a small piece of the cataract is sliced off and tested. The trial judge holds a pre-trial hearing, at which both Dr. Conway and Overton's experts testify. After the hearing, the judge is convinced that Dr. Conway's method of diagnosing cataracts is reliable, because it has been tested and has low error rates and has undergone some peer review. However, the judge also determines that Dr. Conway's method is not generally accepted by the scientific community and there are no standards for this method. Should the trial judge admit Dr. Conway's testimony?

Yes, because the federal courts apply the Daubert standard and this method passes the Daubert standard. Correct. All federal courts apply the Daubert standard. Under the Daubert standard, the trial judge balances a number of factors, and if the trial judge believes the evidence is reliable, the evidence is admissible. The fact pattern made it clear that the trial judge applied the Daubert standard and that the judge found the evidence to be reliable. Under Daubert, these factors are not exclusive, nor does the evidence have to pass every singled Daubert factor to pass the Daubert test.

Draco Chemical's pipeline burst and spilled toxic chemicals into a residential neighborhood. Two separate homeowners, Thomas and Darlene, sued Draco Chemical for damages arising from the spill. The cases were severed and proceeded to trial separately. In the first case, when Thomas was the plaintiff, Draco called its chief engineer to the stand to testify. After the first case was over, the chief engineer was hit by a car and died. The court is now trying the second case, in which Darlene is suing Draco. Draco wants to admit the testimony of its chief engineer as evidence in its defense. Is the chief engineer's testimony admissible?

Yes, but only if Thomas had a similar motive to cross-examine the chief engineer in his case as Darlene does now in her case. Correct. The evidence is hearsay, but it may be admissible under Rule 804(b)(1) as former testimony. For the most part, 804(b)(1) only applies if the opposing party had an opportunity to cross-examine the declarant on a prior occasion, but 804(b)(1)(B) provides that in a civil case, the former testimony is admissible if a "predecessor in interest" had an opportunity and a similar motive to cross-examine the declarant. In this case, Thomas had the opportunity to cross-examine the chief engineer, so if he also had a similar motive to cross-examine as Darlene now has (which is likely), the testimony is admissible against Darlene.

Oswald is accused of sexually assaulting his 17-year-old stepdaughter. The prosecutor calls a doctor to the stand to testify that she examined the stepdaughter one week after the alleged assault and found abrasions to the girl's genitals which are consistent with forcible intercourse. The defendant seeks to admit evidence that a few days before this incident, the stepdaughter had been sexually assaulted by a 16-year-old who was living with her in a foster home at the time. Is this evidence admissible?

Yes, but only if the abrasions could have been caused by the earlier incident. Correct. Rule 412 generally bars any evidence of prior sexual conduct on the part of the victim, but Rule 412(b)(1)(A) allows for such evidence if offered to prove that someone other than the defendant was the source of physical evidence.

The Securities and Exchange Commission ("SEC") is charging Delta Securities with insider trading. The SEC argues that over the course of three years, Delta made thousands of stock purchases and sales that profited off information that its employees had learned through improper means. At trial, the prosecutor seeks to introduce three graphs and five tables which summarize the thousands of transactions that the SEC had monitored over the past three years. Are the graphs and tables admissible?

Yes, but only if the prosecutor has made the underlying data available to Delta in advance of trial so that Delta can examine or copy the data. The underlying data must be admissible, but need not be admitted at trial. Correct. Rule 1006 allows a party to introduce a summary, chart, or calculation to prove the content of "voluminous writings." The underlying data need not be admitted in court, but they must be made available at a reasonable time to the opposing party for examination and copying.

Officer Jackson arrested William after William sold heroin to an undercover officer. Officer Jackson searched William pursuant to the arrest, and found a cell phone in William's pocket. The cell phone had a recent text message in plain view on the front: "How much stuff did you sell tonight?" Assume there are no Fourth Amendment problems with Officer Jackson reading the text. Officer Jackson now wants to testify at William's trial and tell the jury about the text message he read off the cell phone. Is Officer Jackson's testimony about the text message admissible?

Yes, it is admissible as evidence that William was selling things that night. Correct. The statement is not hearsay, because the declarant was not trying to communicate any fact to anyone. Instead, we are inferring a fact about William (that he was selling something that night) from the fact that a third party expected him to be selling something that night.

The prosecutor has charged Steven with aggravated assault on Andy. Steven is arguing self-defense, and Steven has an eyewitness named Sarah who was with Andy on the street a few minutes before the incident. Sarah will testify that she saw Andy pull out a knife and say: "I am going to slash Steven's face to teach him a lesson." Sarah then left the scene and saw nothing else. Should Sarah be allowed to testify in this case?

Yes, she can testify about seeing the knife and she can report what Andy said. Correct. Sarah can testify about everything she saw and everything that Andy said (since it was a statement of intent and therefore admissible under the state of mind exception of 803(3)).

When filing last year's tax return, John understated his income by $20,000 and therefore paid approximately $5,000 less in taxes. John was indicted for intentionally making a false statement on his federal income taxes. During the trial, the prosecutor offered into evidence a letter to the local newspaper that John wrote last year, in which he stated in part: "The United States military buildup is immoral, and every free-thinking citizen should withhold whatever portion of their taxes is being used to pay the defense budget." John objects to this letter. Is John's letter admissible?

Yes. Correct. John's letter proves his motive for under-reporting his taxes, which is relevant because one of the elements of the crime is that he intentionally under-reported. The fact that he had a reason for under-reporting is relevant to prove that he acted intentionally. The statement is not hearsay because it is not being used to prove the truth of the matter asserted (and at any rate, it would be a party-opponent statement). It is not unfairly prejudicial because it has a significant probative value which is not substantially outweighed by unfair prejudice, and it is not likely to mislead or confuse the jury.

Pursuant to a valid court order, the FBI installed a wiretap on a telephone in a warehouse which they suspected was being used to store stolen property. The FBI recorded over fifty hours of conversation between the owner of the warehouse and another man who was referred to only as "Slick." During the course of the conversation, the warehouse owner mentioned a number of stolen items and Slick made offers to purchase them. Other evidence led the FBI to believe that "Slick" was actually Sam Gordon, a known criminal who sold stolen goods over the internet. The FBI agents running the wiretap called in Special Agent Donovan, who had used Gordon as an informant numerous times and was familiar with his voice. Donovan confirmed that the voice of Slick on the tape belonged to Sam Gordon. Gordon was ultimately arrested and charged with receiving stolen property. At trial, the prosecutor seeks to play some portions of the tape to the jury. The prosecutor intends on authenticating the tape by calling Special Agent Donovan to testify that the voice of the man called Slick belongs to Sam Gordon. Gordon will testify that the voice on the tape does not belong to him. Should the tape be admitted into evidence?

Yes. Agent Donovan's testimony is sufficient to authenticate the voice as Gordon's. Correct. Special Agent Donovan's testimony identifying the voice is sufficient to support a prima facie case that the voice on the tape belongs to Sam Gordon. Rule 901(5) states that a witness can give an opinion about the owner of a voice as long as the witness heard the voice at any time under circumstances that connect it with the alleged speaker. Although there is a dispute about this fact, the tape is authenticated unless no rational juror could conclude that the voice belonged to Gordon.

Jerry is suing his former employer, Galaxy Demolition, Inc. Jerry's job at Galaxy was to help tear down abandoned buildings and cart the waste to the local dump. Jerry contends that Galaxy assigned him to tear down an old building with asbestos inside the walls, and that Galaxy did not warn him about the asbestos or give him any protective equipment. Galaxy's defense is that any asbestos that Jerry might have been exposed to was a negligible amount and could not have caused any harm to Jerry. In his case-in-chief, Jerry wants to call his doctor, Dr. Walsh, to the stand. Dr. Walsh will testify that Jerry came to his office on a certain date and said he was having trouble breathing and that he was coughing up blood. Dr. Walsh will tell the jury that he referred Jerry to a specialist named Dr. Greenbaum. Unfortunately Dr. Greenbaum is not available to testify, but Jerry will testify that when he visited Dr. Greenbaum, the doctor performed some tests and then told Jerry that Jerry suffered from asbestos poisoning. Are there any hearsay problems with Dr. Walsh's testimony or Jerry's testimony?

Yes. Jerry should not be permitted to testify about what Dr. Greenbaum told him, since that is hearsay. Correct. Information that a doctor gives to a patient is not covered by Rule 803(4), since it is not information given for the purpose of receiving a medical diagnosis or treatment. If the information is written down as part of a medical record, it is frequently admissible as a business record under Rule 803(6), but this was an oral statement made to a patient.


Kaugnay na mga set ng pag-aaral

Intro. to Programming Ch. 4 practice test

View Set

ECON 2302 Final Micro Tanter Tarleton

View Set